Как найти сопротивление параллельно соединенных резисторов: Онлайн-калькулятор расчета последовательного и параллельного соединения резисторов

Содержание

формула и примеры расчета сопротивления, напряжения, тока и мощности

При проектировании электрических схем возникает необходимость использования последовательного и параллельного соединений резисторов. Соединения применяются также и при ремонтах электрооборудования, поскольку в некоторых ситуациях невозможно найти эквивалентный номинал резистора. Выполнить расчет просто, и справиться с этой операцией может каждый.

Типы проводников

Проводимость веществом электрического тока связана с наличием в нем свободных носителей заряда. Их количество определяется по электронной конфигурации. Для этого необходима химическая формула вещества, при помощи которой можно вычислить их общее число. Значение для каждого элемента берется из периодической системы Дмитрия Ивановича Менделеева.

Электрический ток — упорядоченное движение свободных носителей заряда, на которые воздействует электромагнитное поле. При протекании тока по веществу происходит взаимодействие потока заряженных частиц с узлами кристаллической решетки, при этом часть кинетической энергии частицы превращается в тепловую энергию.

Иными словами, частица «ударяется» об атом, а затем снова продолжает движение, набирая скорость под действием электромагнитного поля.

Процесс взаимодействия частиц с узлами кристаллической решетки называется электрической проводимостью или сопротивлением материала. Единицей измерения является Ом, а определить его можно при помощи омметра или расчитать. Согласно свойству проводимости, вещества можно разделить на 3 группы:

  1. Проводники (все металлы, ионизированный газ и электролитические растворы).
  2. Полупроводники (Si, Ge, GaAs, InP и InSb).
  3. Непроводники (диэлектрики или изоляторы).

Проводники всегда проводят электрический ток, поскольку содержат в своем атомарном строении свободные электроны, анионы, катионы и ионы. Полупроводники проводят электричество только при определенных условиях, которые влияют на наличие или отсутствие свободных электронов и дырок. К факторам, влияющим на проводимость, относятся следующие: температура, освещенность и т. д. Диэлектрики вообще не проводят электричество, поскольку в их структуре вообще отсутствуют свободные носители заряда. При выполнении расчетов каждый радиолюбитель должен знать зависимость сопротивления от некоторых физических величин.

Зависимость сопротивления

Значение электропроводимости зависит от нескольких факторов, которые необходимо учитывать при расчетах, изготовлении элементов резистивной нагрузки (резисторов), ремонте и проектировании устройств. К этим факторам необходимо отнести следующие:

  1. Температура окружающей среды и материала.
  2. Электрические величины.
  3. Геометрические свойства вещества.
  4. Тип материала, из которого изготовлен проводник (полупроводник).

К электрическим величинам можно отнести разность потенциалов (напряжение), электродвижущую силу (ЭДС) и силу тока. Геометрией проводника является его длина и площадь поперечного сечения.

Электрические величины

Зависимость величины электропроводимости от параметров электричества определяется законом Ома. Существует две формулировки: одна — для участка, а другая — для полной цепи. В первом случае соотношение определяются, исходя из значений силы тока (I) и напряжения (U) простой формулой: I = U / R. Из соотношения видна прямо пропорциональная зависимость тока от величины напряжения, а также обратно пропорциональная от сопротивления. Можно выразить R: R = U / I.

Для расчета электропроводимости всего участка следует воспользоваться соотношением между ЭДС (e), силой тока (i), а также внутренним сопротивлением источника питания (Rвн): i = e / (R+Rвн). В этом случае величина R вычисляется по формуле: R = (e / i) — Rвн. Однако при выполнении расчетов необходимо учитывать также геометрические параметры и тип проводника, поскольку они могут существенно повлиять на вычисления.

Тип и геометрические параметры

Свойство вещества к проводимости электричества определяется структурой кристаллической решетки, а также количеством свободных носителей. Исходя из этого, тип вещества является ключевым фактором, который определяет величину электропроводимости. В науке коэффициент, определяющий тип вещества, обозначается литерой «р» и называется удельным сопротивлением. Его значение для различных материалов (при температуре +20 градусов по Цельсию) можно найти в специальных таблицах.

Иногда для удобства расчетов используется обратная величина, которая называется удельной проводимостью (σ). Она связана с удельным сопротивлением следующим соотношением: p = 1 / σ. Площадь поперечного сечения (S) влияет на электрическое сопротивление. С физической точки зрения, зависимость можно понять следующим образом: при малом сечении происходят более частые взаимодействия частиц электрического тока с узлами кристаллической решетки.

Поперечное сечение можно вычислить по специальному алгоритму:

  1. Измерение геометрических параметров проводника (диаметр или длину сторон) при помощи штангенциркуля.
  2. Визуально определить форму материала.
  3. Вычислить площадь поперечного сечения по формуле, найденной в справочнике или интернете.

В случае когда проводник имеет сложную структуру, необходимо вычислить величину S одного элемента, а затем умножить результат на количество элементов, входящих в его состав. Например, если провод является многожильным, то следует вычислить S для одной жилы. После этого нужно умножить, полученную величину S, на количество жил. Зависимость R от вышеперечисленных величин можно записать в виде соотношения: R = p * L / S. Литера «L» является длиной проводника. Однако для получения точных расчетов необходимо учитывать температурные показатели внешней среды и проводника.

Температурные показатели

Существует доказательство зависимости удельного сопротивления материала от температуры, основанное на физическом эксперименте.

Для проведения опыта нужно собрать электрическую цепь, состоящую из следующих элементов: источника питания, нихромовой спирали, соединительных проводов амперметра и вольтметра. Приборы нужны для измерения значений силы тока и напряжения соответственно. При протекании электричества происходит нагревание нихромовой пружины. По мере ее нагревания, показания амперметра уменьшаются. При этом происходит существенное падение напряжения на участке цепи, о котором свидетельствуют показания вольтметра.

В радиотехнике уменьшение величины напряжение называется просадкой или падением. Формула зависимости р от температуры имеет следующий вид: p = p0 * [1 + a * (t — 20)]. Значение p0 — удельное сопротивление материала, взятого из таблицы, а литера «t» — температура проводника.

Температурный коэффициент «а» принимает следующие значения: для металлов — a>0, а для электролитических растворов — a<0. Для получения формулы, определяющей все зависимости, необходимо подставить все соотношения в общую формулу зависимости R от типа материала, температуры, длины и сечения: R = p0 * [1 + a * (t — 20)] * L / S. Формулы используются только для расчетов и изготовления резисторов. Для быстрого измерения величины сопротивления применяется омметр.

Объединение резистивных радиокомпонентов

Для получения необходимого номинала сопротивления применяются два типа соединения резисторов: параллельное и последовательное. Если их соединить параллельно, то нужно два вывода одного резистора подключить к двум выводам другого. Если соединение является последовательным, то один вывод резистора соединяется с одним выводом другого резистора. Соединения используются для получения необходимых номиналов сопротивлений, а также для увеличения рассеивания мощности тока, протекающего по цепи.

Каждое из соединений обладает определенными характеристиками. Кроме того, последовательно или параллельно могут объединяться несколько резисторов. Соединения также могут быть смешанными, т. е.

применяться оба типа объединения радиокомпонентов.

Параллельное соединение

При параллельном подключении значение напряжения на всех резисторах одинаковое, а сила тока — обратно пропорциональна их общему сопротивлению. В интернете web-разработчики создали для расчета величины общего сопротивления параллельного соединения резисторов онлайн-калькулятор.

Рассчитывается общее сопротивление при параллельном соединении по формуле: 1 / Rобщ = (1 / R1) + (1 / R2) + …+ (1 / Rn). Если выполнить математические преобразования и привести к общему знаменателю, то получится удобная формула параллельного соединения для расчета Rобщ. Она имеет следующий вид: Rобщ = (R1 * R2 * … * Rn) / (R1 + R2 + … + Rn). Если необходимо рассчитать величину Rобщ только для двух радиокомпонентов, то формула параллельного сопротивления имеет следующий вид: Rобщ = (R1 * R2) / (R1 + R2).

При ремонте или проектировании схемы устройства возникает задача объединения нескольких резистивных элементов для получения конкретной величины сопротивления. Например, значение Rобщ для определенной цепочки элементов равно 8 Ом, которое получено при расчетах. Перед радиолюбителем стоит задача, какие нужно подобрать номиналы для получения нужного значения (в стандартном ряду резисторов отсутствует радиокомпонент с номиналом в 8 Ом, а только 7,5 и 8,2). В этом случае нужно найти сопротивление при параллельном соединении резистивных элементов. Посчитать значение Rобщ для двух элементов можно следующим образом:

  1. Номинал резистора в 16 Ом подойдет.
  2. Подставить в формулу: R = (16 * 16) / (16 + 16) = 256 / 32 = 8 (Ом).

В некоторых случаях следует потратить больше времени на подбор необходимых номиналов. Можно применять не только два, но и три элемента. Сила тока вычисляется с использованием первого закона Кирхгофа. Формулировка закона следующая: общее значение тока, входящего и протекающего по цепи, равен выходному его значению. Величина силы тока для цепи, состоящей из двух резисторов (параллельное соединение) рассчитывается по такому алгоритму:

  1. Ток, протекающий через R1 и R2: I1 = U / R1 и I2 = U / R2 соответственно.
  2. Общий ток — сложение токов на резисторах: Iобщ = I1 + I2.

Например, если цепь состоит из 2 резисторов, соединенных параллельно, с номиналами в 16 и 7,5 Ом. Они запитаны от источника питания напряжением в 12 В. Значение силы тока на первом резисторе вычисляется следующим способом: I1 = 12 / 16 = 0,75 (А). На втором резисторе ток будет равен: I2 = 12 / 7,5 = 1,6 (А). Общий ток определяется по закону Кирхгофа: I = I1 + I2 = 1,6 + 0,75 = 2,35 (А).

Последовательное подключение

Последовательное включение резисторов также применяется в радиотехнике. Методы нахождения общего сопротивления, напряжения и тока отличаются от параллельного подключения. Основные правила соединения следующие:

  1. Ток не изменяется на участке цепи.
  2. Общее напряжение равно сумме падений напряжений на каждом резисторе.
  3. Rобщ = R1 + R2 + … + Rn.

Пример задачи следующий: цепочка, состоящая из 2 резисторов (16 и 7,5 Ом), питается от источника напряжением 12 В и током в 0,5 А. Необходимо рассчитать электрические параметры для каждого элемента. Порядок расчета следующий:

  1. I = I1 = I2 = 0,5 (А).
  2. Rобщ = R1 + R2 = 16 + 7,5 = 23,5 (Ом).
  3. Падения напряжения: U1 = I * R1 = 0,5 * 16 = 8 (В) и U2 = I * R2 = 0,5 * 7,5 = 3,75 (В).

Не всегда выполняется равенство напряжений (12 В не равно 8 + 3,75 = 11,75 В), поскольку при этом расчете не учитывается сопротивление соединительных проводов. Если схема является сложной, и в ней встречается два типа соединений, то нужно выполнять расчеты по участкам. В первую очередь, рассчитать для параллельного соединения, а затем для последовательного.

Таким образом, параллельное и последовательное соединения резисторов применяются для получения более точных значений сопротивлений, а также при отсутствии необходимого номинала радиокомпонента при проектировании или ремонте устройств.

Калькулятор соединения резисторов онлайн.

Параллельное соединение резисторов

В каждой электрической схеме присутствует резистор, имеющий сопротивление электрическому току. Резисторы бывают двух типов: постоянные и переменные. Во время разработки любой электрической схемы и ремонта электронных изделий часто приходится применять резистор, обладающий необходимым номиналом.

Несмотря на то что для резисторов предусмотрены различные номиналы , может случиться так, что не будет возможности найти необходимый или же вообще ни один элемент не сможет обеспечить требуемый показатель.

Решением этой проблемы может стать применение последовательного и параллельного соединения. Ознакомившись с этой статьей, вы узнаете об особенностях выполнения расчета и подбора различных номиналов сопротивлений.

Параллельное соединение: общая информация

Часто при изготовлении какого-либо устройства используют резисторы, которые соединяются в соответствии с последовательной схемой. Эффект от применения такого варианта сборки сводится к увеличению общего сопротивления цепи. Для данного варианта соединения элементов создаваемое ими сопротивление рассчитывается как сумма номиналов. Если же сборка деталей выполняется по параллельной схеме, то здесь потребуется рассчитать сопротивление , используя нижеописанные формулы.

К схеме параллельного соединения прибегают в ситуации, когда стоит задача по снижению суммарного сопротивления, а, помимо этого, увеличения мощности для группы элементов, подключенных по параллельной схеме, которое должно быть больше, чем при их отдельном подключении.

Расчет сопротивления

В случае подключения деталей друг с другом, с применением параллельной схемы для расчета суммарного сопротивления, будет использоваться следующая формула:

R(общ)=1/(1/R1+1/R2+1/R3+1/Rn).

  • R1- R3 и Rn – резисторы, подсоединенные по параллельной схеме.

Причем, если цепь создается на основе только двух элементов, то для определения суммарного номинального сопротивления следует использовать такую формулу:

R(общ)=R1*R2/R1+R2.

  • R(общ) – суммарное сопротивление;
  • R1 и R2 – резисторы, подсоединенные по параллельной схеме.

Видео: Пример расчёта сопротивления

Универсальная схема расчета

Применительно к радиотехнике следует уделить внимание одному важному правилу: если подключаемые друг к другу элементы по параллельной схеме имеют одинаковый показатель , то для расчета суммарного номинала необходимо общее значение разделить на число подключенных узлов:

  • R(общ) – суммарное значение сопротивления;
  • R – номинал резистора, подсоединенного по параллельной схеме;
  • n – число подключенных узлов.

Особое внимание следует обратить на то, что конечный показатель сопротивления в случае использования параллельной схемы подключения обязательно будет меньше по сравнению с номиналом любого элемента, подключаемого в цепь.

Пример расчёта

Для большей наглядности можно рассмотреть следующий пример: допустим, у нас есть три резистора, чьи номиналы соответственно равны 100, 150 и 30 Ом. Если воспользоваться первой формулой для определения общего номинала, то получим следующее:

R(общ)=1/(1/100+1/150+1/30)=

1/(0,01+0,007+0,03)=1/0,047=21,28Ом.

Если выполнить несложные расчеты, то можно получить следующее: для цепи, включающей в себя три детали, где наименьший показатель сопротивления составляет 30 Ом, результирующее значение номинала будет равно 21,28 Ом. Этот показатель будет меньше минимального значения номинала в цепи практически на 30%.

Важные нюансы

Обычно для резисторов параллельное соединение применяется тогда, когда стоит задача по созданию сопротивления большей мощности. Для ее решения потребуются резисторы, которые должны иметь равные показатели сопротивления и мощности. При таком варианте определить общую мощность можно следующим образом : мощность одного элемента необходимо перемножить с суммарным числом всех резисторов, из которых состоит цепь, подсоединенных друг с другом в соответствии с параллельной схемой.

Скажем, если нами будут использоваться пять резисторов, чей номинал составляет 100 Ом, а мощность каждого равна 1 Вт, которые присоединены друг к другу в соответствии с параллельной схемой, то суммарный показатель сопротивления будет равен 20 Ом, а мощность составит 5 Вт.

Если взять те же резисторы, но подсоединить их в соответствии с последовательной схемой, то конечная мощность составит 5 Вт, а суммарный номинал будет равен 500 Ом.

Видео: Правильное подключение светодиодов

Параллельная схема подключения резисторов очень востребована по той причине, что часто возникает задача по созданию такого номинала, которого невозможно добиться при помощи простого параллельного соединения. При этом процедура расчета этого параметра отличается достаточной сложностью , где необходимо учитывать разные параметры.

Здесь важная роль отводится не только количеству подключаемых элементов, но и рабочим параметрам резисторов — прежде всего, сопротивлению и мощности. Если один из подключаемых элементов будет иметь неподходящий показатель, то это не позволит эффективно решить задачу по созданию требуемого номинала в цепи.

Параллельное соединение резисторов — одно из двух видов электрических соединений, когда оба вывода одного резистора соединены с соответствующими выводами другого резистора или резисторов. Зачастую или параллельно для того, чтобы создать более сложные электронные схемы.

Схема параллельного соединения показан на рисунке ниже. При параллельном соединении резисторов, напряжение на всех резисторах будет одинаковым, а протекающий через них ток будет пропорционален их сопротивлению:

Формула параллельного соединения резисторов

Общее сопротивление нескольких резисторов соединенных параллельно определяется по следующей формуле:

Ток, протекающий через отдельно взятый резистор, согласно , можно найти по формуле:

Параллельное соединение резисторов — расчет

Пример №1

При разработке устройства, возникла необходимость установить резистор с сопротивлением 8 Ом. Если мы просмотрим весь номинальный ряд стандартных значений резисторов, то мы увидим, что резистора с сопротивлением в 8 Ом в нем нет.

Выходом из данной ситуации будет использование двух параллельно соединенных резисторов. Эквивалентное значение сопротивления для двух резисторов соединенных параллельно рассчитывается следующим образом:

Данное уравнение показывает, что если R1 равен R2, то сопротивление R составляет половину сопротивления одного из двух резисторов. При R = 8 Ом, R1 и R2 должны, следовательно, иметь значение 2 × 8 = 16 Ом.
Теперь проведем проверку, рассчитав общее сопротивление двух резисторов:

Таким образом, мы получили необходимое сопротивление 8 Ом, соединив параллельно два резистора по 16 Ом.

Пример расчета №2

Найти общее сопротивление R из трех параллельно соединенных резисторов:

Общее сопротивление R рассчитывается по формуле:

Этот метод расчета может быть использованы для расчета любого количества отдельных сопротивлений соединенных параллельно.

Один важный момент, который необходимо запомнить при расчете параллельно соединенных резисторов – это то, что общее сопротивление всегда будет меньше, чем значение наименьшего сопротивления в этой комбинации.

Как рассчитать сложные схемы соединения резисторов

Более сложные соединения резисторов могут быть рассчитаны путем систематической группировки резисторов. На рисунке ниже необходимо посчитать общее сопротивление цепи, состоящей из трех резисторов:


Для простоты расчета, сначала сгруппируем резисторы по параллельному и последовательному типу соединения.
Резисторы R2 и R3 соединены последовательно (группа 2). Они в свою очередь соединены параллельно с резистором R1 (группа 1).

Последовательное соединение резисторов группы 2 вычисляется как сумма сопротивлений R2 и R3:

В результате мы упрощаем схему в виде двух параллельных резисторов. Теперь общее сопротивление всей схемы можно посчитать следующим образом:

Расчет более сложных соединений резисторов можно выполнить используя законы Кирхгофа.

Ток, протекающий в цепи параллельно соединенных резисторах

Общий ток I протекающий в цепи параллельных резисторов равняется сумме отдельных токов, протекающих во всех параллельных ветвях, причем ток в отдельно взятой ветви не обязательно должен быть равен току в соседних ветвях.

Несмотря на параллельное соединение, к каждому резистору приложено одно и то же напряжение. А поскольку величина сопротивлений в параллельной цепи может быть разной, то и величина протекающего тока через каждый резистор тоже будет отличаться (по определению закона Ома).

Рассмотрим это на примере двух параллельно соединенных резисторов. Ток, который течет через каждый из резисторов (I1 и I2) будет отличаться друг от друга поскольку сопротивления резисторов R1 и R2 не равны.
Однако мы знаем, что ток, который поступает в цепь в точке «А» должен выйти из цепи в точке «B» .

Первое правило Кирхгофа гласит: «Общий ток, выходящий из цепи равен току входящий в цепь».

Таким образом, протекающий общий ток в цепи можно определить как:

Затем с помощью закона Ома можно вычислить ток, который протекает через каждый резистор:

Ток, протекающий в R1 = U ÷ R1 = 12 ÷ 22 кОм = 0,545 мА

Ток, протекающий в R 2 = U ÷ R2 = 12 ÷ 47 кОм = 0,255 мА

Таким образом, общий ток будет равен:

I = 0,545 мА + 0,255 мА = 0,8 мА

Это также можно проверить, используя закон Ома:

I = U ÷ R = 12 В ÷ 15 кОм = 0,8 мА (то же самое)

где 15кОм — это общее сопротивление двух параллельно соединенных резисторов (22 кОм и 47 кОм)

И в завершении хочется отметить, что большинство современных резисторов маркируются цветными полосками и назначение ее можно узнать .

Параллельное соединение резисторов — онлайн калькулятор

Чтобы быстро вычислить общее сопротивление двух и более резисторов, соединенных параллельно, вы можете воспользоваться следующим онлайн калькулятором:

Подведем итог

Когда два или более резистора соединены так, что оба вывода одного резистора соединены с соответствующими выводами другого резистора или резисторов, то говорят, что они соединены между собой параллельно. Напряжение на каждом резисторе внутри параллельной комбинации одинаковое, но токи, протекающие через них, могут отличаться друг от друга, в зависимости от величины сопротивлений каждого резистора.

Эквивалентное или полное сопротивление параллельной комбинации всегда будет меньше минимального сопротивления резистора входящего в параллельное соединение.

1 мОм = 0,001 Ом. 1 кОм = 1 000 = 10³ Ом. 1 МОм = 1 000 000 = 10⁶ Ом.

Эквивалентное сопротивление R eq группы параллельно соединенных резисторов является величиной, обратной сумме величин, обратно пропорциональных сопротивлениям этих резисторов.

Иными словами, проводимость G параллельно соединенных резисторов равна сумме проводимостей этих резисторов:

Эта формула для R eq и используется в данном калькуляторе для расчетов. Например, общее сопротивление трех резисторов 10, 15 и 20 ом, соединенных параллельно, равно 4.62 Ом:

Если параллельно соединены только два резистора, формула упрощается:

Если имеется n соединенных параллельно одинаковых резисторов R , то их эквивалентное сопротивление будет равно

Отметим, что общее сопротивление группы из любого количества соединенных параллельно резисторов всегда будет меньше, чем наименьшее сопротивление резистора в группе и добавление нового резистора всегда приведет к уменьшению эквивалентного сопротивления.

Отметим также, что все резисторы, соединенные параллельно находятся под одним и тем же напряжением. Однако токи, протекающие через отдельные резисторы, отличаются и зависят от их сопротивления. Общий ток через группу резисторов равен сумме токов в отдельных резисторах.

При соединении нескольких резисторов параллельно всегда нужно учитывать их допуски и рассеиваемую мощность.

Примеры применения параллельного соединения резисторов

Одним из примеров параллельного соединения резисторов является шунт в приборе для измерения токов, которые слишком велики для того, чтобы быть напрямую измеренными прибором, предназначенным для измерения небольших токов или напряжений. Для измерения тока параллельно гальванометру или электронному прибору, измеряющему напряжение, подключается резистор с очень маленьким точно известным сопротивлением, изготовленный из материала со стабильными характеристиками. Этот резистор называется шунтом. Измеряемый ток протекает через шунт. В результате на нем падает небольшое напряжение, которое и измеряется вольтметром. Поскольку падение напряжения пропорционально току, протекающему через шунт с известным и точным сопротивлением, вольтметр, подключенный параллельно шунту, можно проградуировать непосредственно в единицах тока (амперах).

Параллельные и последовательные схемы часто используются для получения точного сопротивления или если резистора с требуемым сопротивлением нет или он слишком дорог, если его приобретать в небольших количествах для массового производства . Например, если устройство содержит много резисторов по 20 кОм и необходим только один резистор 10 кОм. Конечно, несложно найти резистор на 10 кОм. Однако для массового производства иногда бывает лучше поставить два резистора на 20 кОм параллельно, чтобы получить необходимые 10 кОм. Это приведет к снижению себестоимости печатной платы, так как будет снижена оптовая цена компонентов, а также стоимость монтажа, так как будет уменьшено количество типоразмеров элементов, которые должен установить на плату автомат установки компонентов.

Проверим справедливость показанных здесь формул на простом эксперименте.

Возьмём два резистора МЛТ-2 на 3 и 47 Ом и соединим их последовательно. Затем измерим общее сопротивление получившейся цепи цифровым мультиметром. Как видим оно равно сумме сопротивлений резисторов, входящих в эту цепочку.


Замер общего сопротивления при последовательном соединении

Теперь соединим наши резисторы параллельно и замерим их общее сопротивление.


Измерение сопротивления при параллельном соединении

Как видим, результирующее сопротивление (2,9 Ом) меньше самого меньшего (3 Ом), входящего в цепочку. Отсюда вытекает ещё одно известное правило, которое можно применять на практике:

При параллельном соединении резисторов общее сопротивление цепи будет меньше наименьшего сопротивления, входящего в эту цепь.

Что ещё нужно учитывать при соединении резисторов?

Во-первых, обязательно учитывается их номинальная мощность. Например, нам нужно подобрать замену резистору на 100 Ом и мощностью 1 Вт . Возьмём два резистора по 50 Ом каждый и соединим их последовательно. На какую мощность рассеяния должны быть рассчитаны эти два резистора?

Поскольку через последовательно соединённые резисторы течёт один и тот же постоянный ток (допустим 0,1 А ), а сопротивление каждого из них равно 50 Ом , тогда мощность рассеивания каждого из них должна быть не менее 0,5 Вт . В результате на каждом из них выделится по 0,5 Вт мощности. В сумме это и будет тот самый 1 Вт .

Данный пример достаточно грубоват. Поэтому, если есть сомнения, стоит брать резисторы с запасом по мощности.

Подробнее о мощности рассеивания резистора читайте .

Во-вторых, при соединении стоит использовать однотипные резисторы, например, серии МЛТ. Конечно, нет ничего плохого в том, чтобы брать разные. Это лишь рекомендация.

Параллельное соединение резисторов, наряду с последовательным, является основным способом соединения элементов в электрической цепи. Во втором варианте все элементы установлены последовательно: конец одного элемента соединен с началом следующего. В такой схеме сила тока на всех элементах одинаковая, а падение напряжений зависит от сопротивления каждого элемента. В последовательном соединении есть два узла. К одному подсоединены начала всех элементов, а ко второму их концы. Условно для постоянного тока можно обозначить их как плюс и минус, а для переменного как фазу и ноль. Благодаря своим особенностям находит широкое применение в электрических схемах, в том числе и со смешанным соединением. Свойства одинаковы для постоянного и переменного тока.

Расчет общего сопротивления при параллельном соединении резисторов

В отличие от последовательного соединения, где для нахождения общего сопротивления достаточно сложить значение каждого элемента, для параллельного то же самое будет справедливо для проводимости. А так как она обратно пропорциональна сопротивлению, получим формулу, представленную вместе со схемой на следующем рисунке:

Необходимо отметить одну важную особенность расчета параллельного соединения резисторов: общее значение будет всегда меньше, чем самое маленькое из них. Для резисторов справедливо как для постоянного, так и для переменного тока. Катушки и конденсаторы имеют свои особенности.

Сила тока и напряжение

При расчете параллельного сопротивления резисторов необходимо знать, как рассчитать напряжение и силу тока. В этом случае нам поможет закон Ома, определяющий связь между сопротивлением, силой тока и напряжением.

Исходя из первой формулировки закона Кирхгофа, получим, что сумма сходящихся в одном узле токов равна нулю. Направление выбираем по направлению протекания тока. Таким образом, положительным направлением для первого узла можно считать входящий ток от источника питания. А отрицательными будут отходящие из каждого резистора. Для второго узла картина противоположна. Исходя из формулировки закона, получим, что суммарный ток равен сумме токов, проходящих через каждый параллельно соединенный резистор.

Итоговое напряжение же определяется по второму закону Кирхгофа. Оно одинаково для каждого резистора и равно общему. Эта особенность используется для подключения розеток и освещения в квартирах.

Пример расчета

В качестве первого примера приведем расчет сопротивления при параллельном соединении одинаковых резисторов. Сила тока, протекающая через них, будет одинаковой. Пример расчета сопротивления выглядит так:

По этому примеру прекрасно видно, что общее сопротивление ниже в два раза, чем каждое из них. Это соответствует тому, что суммарная сила тока в два раза выше, чем у одного. А также прекрасно соотносится с увеличением проводимости в два раза.

Второй пример

Рассмотрим пример параллельного соединения трех резисторов. Для расчета используем стандартную формулу:

Похожим образом рассчитываются схемы с большим количеством параллельно соединенных резисторов.

Пример смешанного соединения

Для смешанного соединения, например, представленного ниже, расчет будет производиться в несколько этапов.

Для начала последовательные элементы можно условно заменить одним резистором, обладающим сопротивлением, равным сумме двух заменяемых. Далее общее сопротивление считаем тем же способом, что и для предыдущего примера. Данный метод подойдет и для других более сложных схем. Последовательно упрощая схему, можно получить необходимое значение.

Например, если вместо резистора R3 будут подключены два параллельных, потребуется сначала рассчитать их сопротивление, заменив их эквивалентным. А далее то же самое, что и в примере выше.

Применение параллельной схемы

Параллельное соединение резисторов находит свое применение во многих случаях. Последовательное подключение увеличивает сопротивление, а для нашего случая оно уменьшится. Например, для электрической цепи требуется сопротивление в 5 Ом, но есть только резисторы на 10 Ом и выше. Из первого примера мы знаем, что можно получить в два раза меньшее значение сопротивления, если установить два одинаковых резистора параллельно друг другу.

Уменьшить сопротивление можно еще больше, например, если две пары параллельно соединенных резисторов соединить параллельно относительно друг друга. Можно уменьшить сопротивление еще в два раза, если резисторы имеют одинаковое сопротивление. Комбинируя с последовательным соединением, можно получить любое значение.

Второй пример — это использование параллельного подключения для освещения и розеток в квартирах. Благодаря такому подключению напряжение на каждом элементе не будет зависеть от их количества и будет одинаковым.

Еще один пример использования параллельного подключения — это защитное заземление электрооборудования. Например, если человек касается металлического корпуса прибора, на который произойдет пробой, получится параллельное соединения его и защитного проводника. Первым узлом будет место прикосновения, а вторым нулевая точка трансформатора. По проводнику и человеку будет течь разный ток. Величину сопротивления последнего принимают за 1000 Ом, хотя реальное значение зачастую гораздо больше. Если бы не было заземления, весь ток, протекающий в схеме, пошел бы через человека, так как он был бы единственным проводником.

Параллельное соединение может использоваться и для батарей. Напряжение при этом остается прежним, однако в два раза возрастает их емкость.

Итог

При подключении резисторов параллельно, напряжение на них будет одинаковым, а ток равен сумме протекающих через каждый резистор. Проводимость будет ровняться сумме каждого. От этого и получается необычная формула суммарного сопротивления резисторов.

Необходимо учитывать при расчете параллельного соединения резисторов то, что итоговое сопротивление будет всегда меньше самого маленького. Это также можно объяснить суммированием проводимости резисторов. Последняя будет возрастать при добавлении новых элементов, соответственно и проводимость будет уменьшаться.

Последовательное и параллельное соединения источников тока

Решение:
Внутреннее сопротивление элементов

Сопротивление параллельно включенных резисторов

Общая э. д. с. элементов e0=2e Согласно закону Ома для полной цепи

15 Сопротивления резисторов R1 и R2 и э. д. с. ε1 и ε2 источников тока в схеме, изображенной на рис. 127, известны. При какой э.д.с. ε3 третьего источника ток через резистор R3 не течет?

Решение:
Выберем направления токов I1, I2 и I3 через резисторы R1, R2 и R3, указанные на рис. 363. Тогда I3=I1+I2. Разность потенциалов между точками а и b будет равна

Если

Исключая I1 находим

16 Цепь из трех одинаковых последовательно соединенных элементов с э.д.с. ε и внутренним сопротивлением r замкнута накоротко (рис. 128). Какое напряжение покажет вольтметр, подключенный к зажимам одного из элементов?

Решение:
Рассмотрим ту же схему без вольтметра (рис. 364). Из закона Ома для полной цепи находим

Из закона Ома для участка цепи между точками а и b получим

Подключение вольтметра к точкам, разность потенциалов между которыми равна нулю, ничего не может изменить в цепи. Поэтому вольтметр будет показывать напряжение, равное нулю.

17 Источник тока с э.д.с. ε0 включен в схему, параметры которой даны на рис. 129. Найти э.д.с. ε источника тока и направление его подключения к выводам а и b, при которых ток через резистор с сопротивлением R2 не идет.

Решение:
Подключим источник тока к выводам а и b и выберем направления токов, указанные на рис. 365. Для узла е имеем I=I0+I2. При обходе контуров aefb и ecdf по часовой стрелке получим

Используя условие I2 = 0, находим

Знак минус показывает, что полюсы источника тока на рис. 365 нужно поменять местами.

18 Два элемента с одинаковыми э.д.с. ε включены в цепь последовательно. Внешнее сопротивление цепи R = 5 Ом. Отношение напряжения на зажимах первого элемента к напряжению на зажимах второго элемента равно 2/3. Найти внутренние сопротивления элементов r1 и r2, если r1=2r2.

Решение:


19 Два одинаковых элемента с э.д.с. ε=1,5 В и внутренним сопротивлением r = 0,2 Ом замкнуты на резистор, сопротивление которого составляет в одном случае R1=0,2 Oм, В другом — R2 = 20 Ом. Как нужно соединить элементы (последовательно или параллельно) в первом и во втором случаях, чтобы получить наибольший ток в цепи?

Решение:
При параллельном соединении двух элементов внутреннее сопротивление и э.д.с. равны r/2 и ε при последовательном соединении они равны 2r и 2ε. Через резистор R при этом текут токи

Отсюда видно, что I2>I1, если R/2+r<R+r/2, т. е. если r1=r; следовательно, токи при параллельном и последовательном соединениях одинаковы. Во втором случае R2>r.Поэтому ток больше при последовательном соединении.

20 Два элемента с э.д.с. ε1=4В и ε2 = 2В и внутренними сопротивлениями r1 = 0,25 Ом и r2 = 0,75 Ом включены в схему, изображенную на рис. 130. Сопротивления резисторов R1 = 1 Ом и R2 = 3 Ом, емкость конденсатора С=2 мкФ. Найти заряд на конденсаторе.

Решение:


21 К батарее из двух параллельно включенных элементов с э.д.с. ε1 и ε2 и внутренними сопротивлениями r1 и r2 подключен резистор с сопротивлением R. Найти ток I, текущий через резистор R, и токи I1 и I2 в первом и втором элементах. При каких условиях токи в отдельных цепях могут быть равными нулю или изменять свое направление на обратное?

Решение:
Выберем направления токов, указанные на рис. 366. Для узла b имеем I-I1-I2=0. При обходе контуров abef и bcde по часовой стрелке получим

Из этих уравнений находим

Ток I=0 тогда, когда изменена полярность включения одного из элементов и, кроме того, выполнено условие

Ток I1=0 при

а ток I2 = 0 при

Токи I1 и I2 имеют направления, указанные на рис.366, если

Они меняют свое направление при

22 Батарея из n одинаковых аккумуляторов, соединенных в одном случае последовательно, в другом— параллельно, замыкается на резистор с сопротивлением R. При каких условиях ток, текущий через резистор, в обоих случаях будет один и тот же?

Решение:
При n(R-r) = R-r. Если R=r, то число элементов произвольно; если R№r, задача не имеет решения (n=1).

23 Батарея из n = 4 одинаковых элементов с внутренним сопротивлением r=2 Ом, соединенных в одном случае последовательно, в другом — параллельно, замыкается на резистор с сопротивлением R=10Ом. Во сколько раз показание вольтметра н одном случае отличается от показания вольтметра в другом случае? Сопротивление вольтметра велико по сравнению с R и r.

Решение:

где V1 — показание вольтметра при последовательном соединении элементов, V2-при параллельном.

24 Как изменится ток, текущий через резистор с сопротивлением R = 2 Ом, если n =10 одинаковых элементов, соединенных последовательно с этим резистором, включить параллельно ему? Э.д.с. элемента ε = 2 В, его внутреннее сопротивление r = 0,2 Ом.

Решение:


25 Батарея составлена из N=600 одинаковых элементов так, что n групп соединены последовательно и в каждой из них содержится т элементов, соединенных параллельно. Э.д.с. каждого элемента ε = 2 В, его внутреннее сопротивление r = 0,4 Ом. При каких значениях n и m батарея, будучи замкнута на внешнее сопротивление R = 0,6 Ом, отдаст во внешнюю цепь максимальную мощность? Найти при этом ток, текущий через сопротивление R.

Решение:
Общее число элементов N=nm (рис. 367). Ток во внешней цепи

где r/m— внутреннее сопротивление группы из т параллельно соединенных элементов, а nr/m — внутреннее сопротивление n групп, соединенных последовательно. Максимальная мощность отдается во внешнюю цепь при равенстве сопротивления R внутреннему сопротивлению батареи элементов nr/m, т. е.

При этом через сопротивление R течет точек I=46 А.

26 Емкость аккумулятора Qo=80А⋅ч. Найти емкость батареи из n = 3 таких аккумуляторов, включенных последовательно и параллельно.

Решение:
При последовательном соединении через все аккумуляторы батареи течет один и тот же ток, поэтому все они разрядятся в течение одного и того же времени. Следовательно, емкость батареи будет равна емкости каждого аккумулятора:
При параллельном соединении n аккумуляторов через каждый из них течет 1/n часть общего тока; поэтому при том же разрядном токе в общей цепи батареи будет разряжаться в n раз дольше, чем один аккумулятор, т. е. емкость батареи в п раз больше емкости отдельного аккумулятора:

Заметим, однако, что энергия

отдаваемая батареей в цепь, и при последовательном и при параллельном соединении n аккумуляторов в n раз больше энергии, отдаваемой одним аккумулятором. Это происходит потому, что при последовательном соединении э. д. с. батареи в n раз больше э. д. с. одного аккумулятора, а при параллельном соединении э.д.с. батареи остается той же, что и для каждого аккумулятора, но Q увеличивается в n раз.

27 Найти емкость батареи аккумуляторов, включенных по схеме, изображенной на рис.131. Емкость каждого аккумулятора Q0=64 А⋅ч.

Решение:
Каждая группа из пяти аккумуляторов, включенных последовательно, имеет емкость

Три параллельно включенные группы дают общую емкость батареи

28 Мост для измерения сопротивлений сбалансирован так, что ток через гальванометр не идет (рис. 132). Ток в правой ветви I=0,2 А. Найти напряжение V на зажимах источника тока. Сопротивления резисторов R1 = 2 Ом, R2 = 4 Ом, R3 = 1 Ом.

Решение:

29 Найти токи, протекающие в каждой ветви цепи, изображенной на рис. 133. Э.д.с. источников тока ε1 = 6,5 В и ε2 = 3,9 В. Сопротивления резисторов R1=R2=R3=R4=R5=R6=R=10 Ом.

Решение:
Составляем уравнения Кирхгофа в соответствии с направлениями токов, указанными на рис. 133: I1 + I2 — I3 = 0 для узла b;
I3 — I4 — I5 =0 для узла h; I5 — I1 — I6 = 0 для узла f: при этом

Для контура abfg (обход по часовой стрелке),

Для контура bcdh (обход против часовой стрелки) и

для контура hdef (обход по часовой стрелке). Решая эту систему уравнений с учетом, что все сопротивления одинаковы и равны R=10 Ом, получим

Отрицательные значения токов I2, I4 и I6 показывают, что при данных э.д.с. источников и сопротивлениях резисторов эти токи текут в стороны, противоположные указанным на рис. 133.

2 параллельное соединение проводников. Ток проводников в параллельном и последовательном соединении

1. Находят эквивалентное сопротивление участков цепи с параллельным соединением резисторов. Рисунок 2. Последовательное соединение резисторов. Для расчета сопротивления таких соединений, всю цепь разбивают на простейшие участки, из параллельно или последовательно соединенных резисторов.

Этот результат следует из того, что в точках разветвления токов (узлы A и B) в цепи постоянного тока не могут накапливаться заряды. Этот результат справедлив для любого числа параллельно включенных проводников.

На рис. 1.9.3 приведен пример такой сложной цепи и указана последовательность вычислений. Следует отметить, что далеко не все сложные цепи, состоящие из проводников с различными сопротивлениями, могут быть рассчитаны с помощью формул для последовательного и параллельного соединения.

При последовательном соединении проводников сила тока во всех проводниках одинакова. При параллельном соединении падение напряжения между двумя узлами, объединяющими элементы цепи, одинаково для всех элементов.

Т.е чем большее сопротивление резистора, тем большее напряжение на него падает. В результате к одной точке (электрическому узлу) может быть присоединено несколько резисторов. При таком соединении, через каждый резистор потечет отдельный ток. Сила данного тока будет обратно пропорциональна сопротивлению резистора.

Таким образом, при параллельном подсоединении резисторов с разным сопротивлением, общее сопротивление будет всегда меньше значения самого маленького отдельного резистора. Напряжение между точками A и B является как общим напряжением для всего участка цепи, так и напряжением, падающим на каждый резистор в отдельности. Смешанным соединением называют участок цепи, где часть резисторов соединяются между собой последовательно, а часть параллельно.

Цепь разбивают на участки с только пареллельным или только последовательным соединением. Вычисляют общее сопротивление для каждого отдельного участка. Вычисляют общее сопротивление для всей цепи смешанного соединения. Также существует более быстрый способ расчета общего сопротивления для смешанного соединения. Если резисторы соединяются последоватеьно — складывать.

То есть при последовательном соединении резисторы подключатся друг за другом. На рисунке 4 показан простейший пример смешанного соединения резисторов. После расчета эквивалентных сопротивлений резисторов перерисовывают схему. Обычно получается цепь из последовательно соединенных эквивалентных сопротивлений.4. Рисунок 5. Расчет сопротивления участка цепи при смешанном соединении резисторов.

В результате вы научитесь с нуля не тольно разрабатывать собственные устройства, но и сопрягать с ними различную переферию! Узел — точка разветвления цепи, в которой соединяются не менее трёх проводников. Последовательное соединение резисторов применяется для увеличения сопротивления.

Напряжение при параллельном соединении

Как видно, вычислить сопротивление двух параллельных резисторов значительно удобнее. Параллельное соединение резисторов часто используют в случаях, когда необходимо сопротивление с большей мощностью. Для этого, как правило, используют резисторы с одинаковой мощностью и одинаковым сопротивлением.

Общее сопротивление Rобщ

Такое соединение сопротивлений называется последовательным. Мы получили таким образом, что U = 60 В, т. е. несуществующее равенство ЭДС источника тока и его напряжения. Будем теперь включать амперметр поочередно в каждую ветвь цепи, запоминая показания прибора. Следовательно, при параллельном соединении сопротивлений напряжение на зажимах источника тока равно падению напряжения на каждом сопротивлении.

Такое разветвление тока в параллельных ветвях сходно с течением жидкости по трубам. Рассмотрим теперь, чему будет равно общее сопротивление внешней цепи, состоящей из двух параллельно соединенных сопротивлений.

Вернемся к цепи, показанной на рис. 3, и посмотрим, чему будет равно эквивалентное сопротивление двух параллельно соединенных сопротивлений. Точно так же для каждой ветви I1 = U1 / R1, I2 = U2 / R2, где I1 и I2 — токи в ветвях; U1 и U2 — напряжение на ветвях; R1 и R2 — сопротивления ветвей.

Это значит, что общее сопротивление цепи всегда будет ниже любого параллельно включенного резистора. 2. Если эти участки включают последовательно соединенные резисторы, то сначала вычисляют их сопротивление. Применяя закон Ома для участка цепи, можно доказать, что полное сопротивление при последовательном соединении равно сумме сопротивлений отдельных проводников.

Нужно вычислить сопротивление последовательной, параллельной или комбинированной цепей? Нужно, если вы не хотите сжечь плату! Эта статья расскажет вам, как это сделать. Перед чтением, пожалуйста, уясните, что у резисторов нет «начала» и нет «конца». Эти слова вводятся для облегчения понимания изложенного материала.

Шаги

Сопротивление последовательной цепи

Сопротивление параллельной цепи

Сопротивление комбинированной цепи

Некоторые факты

  1. Каждый электропроводный материал имеет некоторое сопротивление, являющееся сопротивляемостью материала электрическому току.
  2. Сопротивление измеряется в Омах. Символ единицы измерения Ом — Ω.
  3. Разные материалы имеют разные значения сопротивления.
    • Например, сопротивление меди 0.0000017 Ом/см 3
    • Сопротивление керамики около 10 14 Ом/см 3
  4. Чем больше значение сопротивления, тем выше сопротивляемость электрическому току. Медь, которая часто используется в электрических проводах, имеет очень малое сопротивление. С другой стороны, сопротивление керамики очень велико, что делает ее прекрасным изолятором.
  5. Работа всей цепи зависит от того, какой тип соединения вы выберете для подключения резисторов в этой цепи.
  6. U=IR. Это закон Ома, установленный Георгом Омом в начале 1800х. Если вам даны любые две из этих переменных, вы легко найдете третью.
    • U=IR: Напряжение (U) есть результат умножения силы тока (I) * на сопротивление (R).
    • I=U/R: Сила тока есть частное от напряжение (U) ÷ сопротивление (R).
    • R=U/I: Сопротивление есть частное от напряжение (U) ÷ сила тока (I).
  • Запомните: при параллельном соединении существует несколько путей прохождения тока по цепи, поэтому в такой цепи общее сопротивление будет меньше сопротивления каждого отдельного резистора. При последовательном соединении ток проходит через каждый резистор в цепи, поэтому сопротивление каждого отдельного резистора добавляется к общему сопротивлению.
  • Общее сопротивление в параллельной цепи всегда меньше сопротивления одного резистора с самым низким сопротивлением в этой цепи. Общее сопротивление в последовательной цепи всегда больше сопротивления одного резистора с самым высоким сопротивлением в этой цепи.

Одним из китов, на котором держатся многие понятия в электронике, является понятие последовательного и параллельного подключения проводников. Знать основные отличия указанных типов подключения просто необходимо. Без этого нельзя понять и прочитать ни одной схемы.

Основные принципы

Электрический ток движется по проводнику от источника к потребителю (нагрузке). Чаще всего в качестве проводника выбирается медный кабель. Связано это с требованием, которое предъявляется к проводнику: он должен легко высвобождать электроны.

Независимо от способа подключения, электрический ток двигается от плюса к минусу. Именно в этом направлении убывает потенциал. При этом стоит помнить, что провод, по котору идет ток, также обладает сопротивлением. Но его значение очень мало. Именно поэтому им пренебрегают. Сопротивление проводника принимают равным нулю. В том случае, если проводник обладает сопротивлением, его принято называть резистором.

Параллельное подключение

В данном случае элементы, входящие в цепь, объединены между собой двумя узлами. С другими узлами у них связей нет. Участки цепи с таким подключением принято называть ветвями. Схема параллельного подключения представлена на рисунке ниже.

Если говорить более понятным языком, то в данном случае все проводники одним концом соединены в одном узле, а вторым — во втором. Это приводит к тому, что электрический ток разделяется на все элементы. Благодаря этому увеличивается проводимость всей цепи.

При подключении проводников в цепь данным способом напряжение каждого из них будет одинаково. А вот сила тока всей цепи будет определяться как сумма токов, протекающих по всем элементам. С учетом закона Ома путем нехитрых математических расчетов получается интересная закономерность: величина, обратная общему сопротивлению всей цепи, определяется как сумма величин, обратных сопротивлениям каждого отдельного элемента. При этом учитываются только элементы, подключенные параллельно.

Последовательное подключение

В данном случае все элементы цепи соединены таким образом, что они не образуют ни одного узла. При данном способе подключения имеется один существенный недостаток. Он заключается в том, что при выходе из строя одного из проводников все последующие элементы работать не смогут. Ярким примером такой ситуации является обычная гирлянда. Если в ней перегорает одна из лампочек, то вся гирлянда перестает работать.

Последовательное подключение элементов отличается тем, что сила тока во всех проводниках равна. Что касается напряжения цепи, то оно равно сумме напряжения отдельных элементов.

В данной схеме проводники включаются в цепь поочередно. А это значит, что сопротивление всей цепи будет складываться из отдельных сопротивлений, характерных для каждого элемента. То есть общее сопротивление цепи равно сумме сопротивлений всех проводников. Эту же зависимость можно вывести и математическим способом, используя закон Ома.

Смешанные схемы

Бывают ситуации, когда на одной схеме можно увидеть одновременно последовательное и параллельное подключение элементов. В таком случае говорят о смешанном соединении. Расчет подобных схем проводится отдельно для каждой из группы проводников.

Так, чтобы определить общее сопротивление, необходимо сложить сопротивление элементов, подключенных параллельно, и сопротивление элементов с последовательным подключением. При этом последовательное подключение является доминантным. То есть его рассчитывают в первую очередь. И только после этого определяют сопротивление элементов с параллельным подключением.

Подключение светодиодов

Зная основы двух типов подключения элементов в цепи, можно понять принцип создания схем различных электроприборов. Рассмотрим пример. во многом зависит от напряжения источника тока.

При небольшом напряжении сети (до 5 В) светодиоды подключают последовательно. Снизить уровень электромагнитных помех в данном случае поможет конденсатор проходного типа и линейные резисторы. Проводимость светодиодов увеличивают за счет использования системных модуляторов.

При напряжении сети 12 В может использоваться и последовательное, и параллельное подключение сети. В случае последовательного подключения используют импульсные блоки питания. Если собирается цепь из трех светодиодов, то можно обойтись без усилителя. Но если цепь будет включать большее количество элементов, то усилитель необходим.

Во втором случае, то есть при параллельном подключении, необходимо использование двух открытых резисторов и усилителя (с пропускной способностью выше 3 А). Причем первый резистор устанавливается перед усилителем, а второй — после.

При высоком напряжении сети (220 В) прибегают к последовательному подключению. При этом дополнительно используют операционные усилители и понижающие блоки питания.

Содержание:

Течение тока в электрической цепи осуществляется по проводникам, в направлении от источника к потребителям. В большинстве подобных схем используются медные провода и электрические приемники в заданном количестве, обладающие различным сопротивлением. В зависимости выполняемых задач, в электрических цепях используется последовательное и параллельное соединение проводников. В некоторых случаях могут быть применены оба типа соединений, тогда этот вариант будет называться смешанным. Каждая схема имеет свои особенности и отличия, поэтому их нужно обязательно заранее учитывать при проектировании цепей, ремонте и обслуживании электрооборудования.

Последовательное соединение проводников

В электротехнике большое значение имеет последовательное и параллельное соединение проводников в электрической цепи. Среди них часто используется схема последовательного соединения проводников предполагающая такое же соединение потребителей. В этом случае включение в цепь выполняется друг за другом в порядке очередности. То есть, начало одного потребителя соединяется с концом другого при помощи проводов, без каких-либо ответвлений.

Свойства такой электрической цепи можно рассмотреть на примере участков цепи с двумя нагрузками. Силу тока, напряжение и сопротивление на каждом из них следует обозначить соответственно, как I1, U1, R1 и I2, U2, R2. В результате, получились соотношения, выражающие зависимость между величинами следующим образом: I = I1 = I2, U = U1 + U2, R = R1 + R2. Полученные данные подтверждаются практическим путем с помощью проведения измерений амперметром и вольтметром соответствующих участков.

Таким образом, последовательное соединение проводников отличается следующими индивидуальными особенностями:

  • Сила тока на всех участках цепи будет одинаковой.
  • Общее напряжение цепи составляет сумму напряжений на каждом участке.
  • Общее сопротивление включает в себя сопротивления каждого отдельного проводника.

Данные соотношения подходят для любого количества проводников, соединенных последовательно. Значение общего сопротивления всегда выше, чем сопротивление любого отдельно взятого проводника. Это связано с увеличением их общей длины при последовательном соединении, что приводит и к росту сопротивления.

Если соединить последовательно одинаковые элементы в количестве n, то получится R = n х R1, где R — общее сопротивление, R1 — сопротивление одного элемента, а n — количество элементов. Напряжение U, наоборот, делится на равные части, каждая из которых в n раз меньше общего значения. Например, если в сеть с напряжением 220 вольт последовательно включаются 10 ламп одинаковой мощности, то напряжение в любой из них составит: U1 = U/10 = 22 вольта.

Проводники, соединенные последовательно, имеют характерную отличительную особенность. Если во время работы отказал хотя-бы один из них, то течение тока прекращается во всей цепи. Наиболее ярким примером является , когда одна перегоревшая лампочка в последовательной цепи, приводит к выходу из строя всей системы. Для установления перегоревшей лампочки понадобится проверка всей гирлянды.

Параллельное соединение проводников

В электрических сетях проводники могут соединяться различными способами: последовательно, параллельно и комбинированно. Среди них параллельное соединение это такой вариант, когда проводники в начальных и конечных точках соединяются между собой. Таким образом, начала и концы нагрузок соединяются вместе, а сами нагрузки располагаются параллельно относительно друг друга. В электрической цепи могут содержаться два, три и более проводников, соединенных параллельно.

Если рассматривать последовательное и параллельное соединение, сила тока в последнем варианте может быть исследована с помощью следующей схемы. Берутся две лампы накаливания, обладающие одинаковым сопротивлением и соединенные параллельно. Для контроля к каждой лампочке подключается собственный . Кроме того, используется еще один амперметр, контролирующий общую силу тока в цепи. Проверочная схема дополняется источником питания и ключом.

После замыкания ключа нужно контролировать показания измерительных приборов. Амперметр на лампе № 1 покажет силу тока I1, а на лампе № 2 — силу тока I2. Общий амперметр показывает значение силы тока, равное сумме токов отдельно взятых, параллельно соединенных цепей: I = I1 + I2. В отличие от последовательного соединения, при перегорании одной из лампочек, другая будет нормально функционировать. Поэтому в домашних электрических сетях используется параллельное подключение приборов.

С помощью такой же схемы можно установить значение эквивалентного сопротивления. С этой целью в электрическую цепь добавляется вольтметр. Это позволяет измерить напряжение при параллельном соединении, сила тока при этом остается такой же. Здесь также имеются точки пересечения проводников, соединяющих обе лампы.

В результате измерений общее напряжение при параллельном соединении составит: U = U1 = U2. После этого можно рассчитать эквивалентное сопротивление, условно заменяющее все элементы, находящиеся в данной цепи. При параллельном соединении, в соответствии с законом Ома I = U/R, получается следующая формула: U/R = U1/R1 + U2/R2, в которой R является эквивалентным сопротивлением, R1 и R2 — сопротивления обеих лампочек, U = U1 = U2 — значение напряжения, показываемое вольтметром.

Следует учитывать и тот фактор, что токи в каждой цепи, в сумме составляют общую силу тока всей цепи. В окончательном виде формула, отражающая эквивалентное сопротивление будет выглядеть следующим образом: 1/R = 1/R1 + 1/R2. При увеличении количества элементов в таких цепях — увеличивается и число слагаемых в формуле. Различие в основных параметрах отличают друг от друга и источников тока, позволяя использовать их в различных электрических схемах.

Параллельное соединение проводников характеризуется достаточно малым значением эквивалентного сопротивления, поэтому сила тока будет сравнительно высокой. Данный фактор следует учитывать, когда в розетки включается большое количество электроприборов. В этом случае сила тока значительно возрастает, приводя к перегреву кабельных линий и последующим возгораниям.

Законы последовательного и параллельного соединения проводников

Данные законы, касающиеся обоих видов соединений проводников, частично уже были рассмотрены ранее.

Для более четкого их понимания и восприятия в практической плоскости, последовательное и параллельное соединение проводников, формулы следует рассматривать в определенной последовательности:

  • Последовательное соединение предполагает одинаковую силу тока в каждом проводнике: I = I1 = I2.
  • параллельное и последовательное соединение проводников объясняет в каждом случае по-своему. Например, при последовательном соединении, напряжения на всех проводниках будут равны между собой: U1 = IR1, U2 = IR2. Кроме того, при последовательном соединении напряжение составляет сумму напряжений каждого проводника: U = U1 + U2 = I(R1 + R2) = IR.
  • Полное сопротивление цепи при последовательном соединении состоит из суммы сопротивлений всех отдельно взятых проводников, независимо от их количества.
  • При параллельном соединении напряжение всей цепи равно напряжению на каждом из проводников: U1 = U2 = U.
  • Общая сила тока, измеренная во всей цепи, равна сумме токов, протекающих по всем проводникам, соединенных параллельно между собой: I = I1 + I2.

Для того чтобы более эффективно проектировать электрические сети, нужно хорошо знать последовательное и параллельное соединение проводников и его законы, находя им наиболее рациональное практическое применение.

Смешанное соединение проводников

В электрических сетях как правило используется последовательное параллельное и смешанное соединение проводников, предназначенное для конкретных условий эксплуатации. Однако чаще всего предпочтение отдается третьему варианту, представляющему собой совокупность комбинаций, состоящих из различных типов соединений.

В таких смешанных схемах активно применяется последовательное и параллельное соединение проводников, плюсы и минусы которых обязательно учитываются при проектировании электрических сетей. Эти соединения состоят не только из отдельно взятых резисторов, но и довольно сложных участков, включающих в себя множество элементов.

Смешанное соединение рассчитывается в соответствии с известными свойствами последовательного и параллельного соединения. Метод расчета заключается в разбивке схемы на более простые составные части, которые считаются отдельно, а потом суммируются друг с другом.

Последовательным называется такое соединение резисторов, когда конец одного проводника соединяется с началом другого и т.д. (рис. 1). При последовательном соединении сила тока на любом участке электрической цепи одинакова. Это объясняется тем, что заряды не могут накапливаться в узлах цепи. Их накопление привело бы к изменению напряженности электрического поля, а следовательно, и к изменению силы тока. Поэтому

Амперметр А измеряет силу тока в цепи и обладает малым внутренним сопротивлением (R A 0).

Включенные вольтметры V 1 и V 2 измеряют напряжение U 1 и U 2 на сопротивлениях R 1 и R 2 . Вольтметр V измеряет подведенное к клеммам М и N напряжение U. Вольтметры показывают, что при последовательном соединении напряжение U равно сумме напряжений на отдельных участках цепи:

Применяя закон Ома для каждого участка цепи, получим:

где R — общее сопротивление последовательно соединенной цепи. Подставляя U, U 1 , U 2 в формулу (1), имеем

Сопротивление цепи, состоящей из n последовательно соединенных резисторов, равно сумме сопротивлений этих резисторов:

Если сопротивления отдельных резисторов равны между собой, т.е. R 1 = R 2 = … = R n , то общее сопротивление этих резисторов при последовательном соединении в n раз больше сопротивления одного резистора: R = nR 1 .

При последовательном соединении резисторов справедливо соотношение

т.е. напряжения на резисторах прямо пропорциональны сопротивлениям.

Параллельным называется такое соединение резисторов, когда одни концы всех резисторов соединены в один узел, другие концы — в другой узел (рис. 2). Узлом называется точка разветвленной цепи, в которой сходятся более двух проводников. При параллельном соединении резисторов к точкам М и N подключен вольтметр. Он показывает, что напряжения на отдельных участках цепи с сопротивлениями R 1 и R 2 равны. Это объясняется тем, что работа сил стационарного электрического поля не зависит от формы траектории:

Амперметр показывает, что сила тока I в неразветвленной части цепи равна сумме сил токов I 1 и I 2 в параллельно соединенных проводниках R 1 и R 2:

Это вытекает и из закона сохранения электрического заряда. Применим закон Ома для отдельных участков цепи и всей цепи с общим сопротивлением R:

Подставляя I, I 1 и I 2 в формулу (2), получим.

При последовательном соединении сопротивление равно. Параллельное соединение сопротивлениий (резисторов)

Последовательное, параллельное и смешанное соединения резисторов. Значительное число приемников, включенных в электрическую цепь (электрические лампы, электронагревательные приборы и др.), можно рассматривать как некоторые элементы, имеющие определенное сопротивление. Это обстоятельство дает нам возможность при составлении и изучении электрических схем заменять конкретные приемники резисторами с определенными сопротивлениями. Различают следующие способы соединения резисторов (приемников электрической энергии): последовательное, параллельное и смешанное.

Последовательное соединение резисторов . При последовательном соединении нескольких резисторов конец первого резистора соединяют с началом второго, конец второго — с началом третьего и т. д. При таком соединении по всем элементам последовательной цепи проходит
один и тот же ток I.
Последовательное соединение приемников поясняет рис. 25, а.
.Заменяя лампы резисторами с сопротивлениями R1, R2 и R3, получим схему, показанную на рис. 25, б.
Если принять, что в источнике Ro = 0, то для трех последовательно соединенных резисторов согласно второму закону Кирхгофа можно написать:

E = IR 1 + IR 2 + IR 3 = I(R 1 + R 2 + R 3) = IR эк (19)

где R эк = R 1 + R 2 + R 3 .
Следовательно, эквивалентное сопротивление последовательной цепи равно сумме сопротивлений всех последовательно соединенных резисторов.Так как напряжения на отдельных участках цепи согласно закону Ома: U 1 =IR 1 ; U 2 = IR 2 , U 3 = IR з и в данном случае E = U, то длярассматриваемой цепи

U = U 1 + U 2 +U 3 (20)

Следовательно, напряжение U на зажимах источника равно сумме напряжений на каждом из последовательно включенных резисторов.
Из указанных формул следует также, что напряжения распределяются между последовательно соединенными резисторами пропорционально их сопротивлениям:

U 1: U 2: U 3 = R 1: R 2: R 3 (21)

т. е. чем больше сопротивление какого-либо приемника в последовательной цепи, тем больше приложенное к нему напряжение.

В случае если последовательно соединяются несколько, например п, резисторов с одинаковым сопротивлением R1, эквивалентное сопротивление цепи Rэк будет в п раз больше сопротивления R1, т. е. Rэк = nR1. Напряжение U1 на каждом резисторе в этом случае в п раз меньше общего напряжения U:

При последовательном соединении приемников изменение сопротивления одного из них тотчас же влечет за собой изменение напряжения на других связанных с ним приемниках. При выключении или обрыве электрической цепи в одном из приемников и в остальных приемниках прекращается ток. Поэтому последовательное соединение приемников применяют редко — только в том случае, когда напряжение источника электрической энергии больше номинального напряжения, на которое рассчитан потребитель. Например, напряжение в электрической сети, от которой питаются вагоны метрополитена, составляет 825 В, номинальное же напряжение электрических ламп, применяемых в этих вагонах, 55 В. Поэтому в вагонах метрополитена электрические лампы включают последовательно по 15 ламп в каждой цепи.
Параллельное соединение резисторов . При параллельном соединении нескольких приемников они включаются между двумя точками электрической цепи, образуя параллельные ветви (рис. 26, а). Заменяя

лампы резисторами с сопротивлениями R1, R2, R3, получим схему, показанную на рис. 26, б.
При параллельном соединении ко всем резисторам приложено одинаковое напряжение U. Поэтому согласно закону Ома:

I 1 =U/R 1 ; I 2 =U/R 2 ; I 3 =U/R 3 .

Ток в неразветвленной части цепи согласно первому закону Кирхгофа I = I 1 +I 2 +I 3 , или

I = U / R 1 + U / R 2 + U / R 3 = U (1/R 1 + 1/R 2 + 1/R 3) = U / R эк (23)

Следовательно, эквивалентное сопротивление рассматриваемой цепи при параллельном соединении трех резисторов определяется формулой

1/R эк = 1/R 1 + 1/R 2 + 1/R 3 (24)

Вводя в формулу (24) вместо значений 1/R эк, 1/R 1 , 1/R 2 и 1/R 3 соответствующие проводимости G эк, G 1 , G 2 и G 3 , получим: эквивалентная проводимость параллельной цепи равна сумме проводимостей параллельно соединенных резисторов :

G эк = G 1 + G 2 +G 3 (25)

Таким образом, при увеличении числа параллельно включаемых резисторов результирующая проводимость электрической цепи увеличивается, а результирующее сопротивление уменьшается.
Из приведенных формул следует, что токи распределяются между параллельными ветвями обратно пропорционально их электрическим сопротивлениям или прямо пропорционально их проводимостям. Например, при трех ветвях

I 1: I 2: I 3 = 1/R 1: 1/R 2: 1/R 3 = G 1 + G 2 + G 3 (26)

В этом отношении имеет место полная аналогия между распределением токов по отдельным ветвям и распределением потоков воды по трубам.
Приведенные формулы дают возможность определить эквивалентное сопротивление цепи для различных конкретных случаев. Например, при двух параллельно включенных резисторах результирующее сопротивление цепи

R эк =R 1 R 2 /(R 1 +R 2)

при трех параллельно включенных резисторах

R эк =R 1 R 2 R 3 /(R 1 R 2 +R 2 R 3 +R 1 R 3)

При параллельном соединении нескольких, например n, резисторов с одинаковым сопротивлением R1 результирующее сопротивление цепи Rэк будет в n раз меньше сопротивления R1, т.е.

R эк = R1 / n (27)

Проходящий по каждой ветви ток I1, в этом случае будет в п раз меньше общего тока:

I1 = I / n (28)

При параллельном соединении приемников, все они находятся под одним и тем же напряжением, и режим работы каждого из них не зависит от остальных. Это означает, что ток, проходящий по какому-либо из приемников, не будет оказывать существенного влияния на другие приемники. При всяком выключении или выходе из строя любого приемника остальные приемники остаются вклю-

ченными. Поэтому параллельное соединение имеет существенные преимущества перед последовательным, вследствие чего оно получило наиболее широкое распространение. В частности, электрические лампы и двигатели, предназначенные для работы при определенном (номинальном) напряжении, всегда включают параллельно.
На электровозах постоянного тока и некоторых тепловозах тяговые двигатели в процессе регулирования скорости движения нужно включать под различные напряжения, поэтому они в процессе разгона переключаются с последовательного соединения на параллельное.

Смешанное соединение резисторов . Смешанным соединением называется такое соединение, при котором часть резисторов включается последовательно, а часть — параллельно. Например, в схеме рис. 27, а имеются два последовательно включенных резистора сопротивлениями R1 и R2, параллельно им включен резистор сопротивлением Rз, а резистор сопротивлением R4 включен последовательно с группой резисторов сопротивлениями R1, R2 и R3.
Эквивалентное сопротивление цепи при смешанном соединении обычно определяют методом преобразования, при котором сложную цепь последовательными этапами преобразовывают в простейшую. Например, для схемы рис. 27, а вначале определяют эквивалентное сопротивление R12 последовательно включенных резисторов с сопротивлениями R1 и R2: R12 = R1 + R2. При этом схема рис. 27, а заменяется эквивалентной схемой рис. 27, б. Затем определяют эквивалентное сопротивление R123 параллельно включенных сопротивлений и R3 по формуле

R 123 =R 12 R 3 /(R 12 +R 3)=(R 1 +R 2)R 3 /(R 1 +R 2 +R 3).

При этом схема рис. 27, б заменяется эквивалентной схемой рис. 27, в. После этого находят эквивалентное сопротивление всей цепи суммированием сопротивления R123 и последовательно включенного с ним сопротивления R4:

R эк = R 123 + R 4 = (R 1 + R 2) R 3 / (R 1 + R 2 + R 3) + R 4

Последовательное, параллельное и смешанное соединения широко применяют для изменения сопротивления пусковых реостатов при пуске э. п. с. постоянного тока.

1. При последовательном соединении проводников

1. Сила тока во всех проводниках одинакова :

I 1 = I 2 = I

2. Общее напряжение U на обоих проводниках равно сумме напряжений U 1 и U 2 на каждом проводнике :

U = U 1 + U 2

3. По закону Ома, напряжения U 1 и U 2 на проводниках равны U 1 = IR 1 , U 2 = IR 2 а общее напряжение U = IR где R – электрическое сопротивление всей цепи, тогда IR = IR 1 + I R 2. Отсюда следует

R = R 1 + R 2

При последовательном соединении полное сопротивление цепи равно сумме сопротивлений отдельных проводников.

Этот результат справедлив для любого числа последовательно соединенных проводников.

2. При параллельном соединении проводников

1. Напряжения U 1 и U 2 на обоих проводниках одинаковы

U 1 = U 2 = U

2. Сумма токов I 1 + I 2 , протекающих по обоим проводникам, равна току в неразветвленной цепи :

I = I 1 + I 2

Этот результат следует из того, что в точках разветвления токов (узлы A и B ) в цепи постоянного тока не могут накапливаться заряды. Например, к узлу A за время Δt подтекает заряд I Δt , а утекает от узла за то же время заряд I 1 Δt + I 2 Δt . Следовательно, I = I 1 + I 2 .

3. Записывая на основании закона Ома

где R – электрическое сопротивление всей цепи, получим

При параллельном соединении проводников величина, обратная общему сопротивлению цепи, равна сумме величин, обратных сопротивлениям параллельно включенных проводников.

Этот результат справедлив для любого числа параллельно включенных проводников.

Формулы для последовательного и параллельного соединения проводников позволяют во многих случаях рассчитывать сопротивление сложной цепи, состоящей из многих резисторов. На рисунке приведен пример такой сложной цепи и указана последовательность вычислений. Сопротивления всех проводников указаны в омах (Ом).


На пракутике одного источника тока в цепи бывает недостаточно, и тогда источники тока тоже соединяют между собой для питания цепи. Соединение источников в батарею может быть последовательным и параллельным.

При последовательном соединении два соседних источника соединяются разноименными полюсами.

Т.е., для последовательного соединения аккумуляторов, к ″плюсу″ электрической схемы подключают положительную клемму первого аккумулятора. К его отрицательной клемме подключают положительную клемму второго аккумулятора и т.д. Отрицательную клемму последнего аккумулятора подключают к ″минусу″ электрической схемы.

Получившаяся при последовательном соединении аккумуляторная батарея имеет ту же емкость, что и у одиночного аккумулятора, а напряжение такой аккумуляторной батареи равно сумме напряжений входящих в нее аккумуляторов. Т.е. если аккумуляторы имеют одинаковые напряжения, то напряжение батареи равно напряжению одного аккумулятора, умноженному на количество аккумуляторов в аккумуляторной батарее.


1. ЭДС батареи равна сумме ЭДС отдельных источников ε= ε 1 + ε 2 + ε 3

2 . Общее сопротивление батареи источников равно сумме внутренних сопротивлений отдельных источников r батареи = r 1 + r 2 + r 3

Если в батарею соединены n одинаковых источников, то ЭДС батареи ε= nε 1, а сопротивление r батареи = nr 1

3.

При параллельном соединении соединяют между собой все положительные и все отрицательные полюсы двух или n источников.

Т.е., при параллельном соединении, аккумуляторы соединяют так, чтобы положительные клеммы всех аккумуляторов были подключены к одной точке электрической схемы (″плюсу″), а отрицательные клеммы всех аккумуляторов были подключены к другой точке схемы (″минусу″).

Параллельно соединяют только источники с одинаковой ЭДС . Получившаяся при параллельном соединении аккумуляторная батарея имеет то же напряжение, что и у одиночного аккумулятора, а емкость такой аккумуляторной батареи равна сумме емкостей входящих в нее аккумуляторов. Т.е. если аккумуляторы имеют одинаковые емкости, то емкость аккумуляторной батареи равна емкости одного аккумулятора, умноженной на количество аккумуляторов в батарее.


1. ЭДС батареи одинаковых источников равна ЭДС одного источника. ε= ε 1 = ε 2 = ε 3

2. Сопротивление батареи меньше, чем сопротивление одного источника r батареи = r 1 /n
3. Сила тока в такой цепи по закону Ома

Электрическая энергия, накопленная в аккумуляторной батарее равна сумме энергий отдельных аккумуляторов (произведению энергий отдельных аккумуляторов, если аккумуляторы одинаковые), независимо от того, как соединены аккумуляторы — параллельно или последовательно.

Внутреннее сопротивление аккумуляторов, изготовленных по одной технологии, примерно обратно пропорционально емкости аккумулятора. Поэтому т.к.при параллельном соединении емкость аккумуляторной батареи равна сумме емкостей входящих в нее аккумуляторов, т.е увеличивается, то внутреннее сопротивление уменьшается.

Параллельным соединением сопротивлений называется такое соединение, когда начала сопротивлений соединены в одну общую точку, а концы — в другую.

Для параллельного соединения сопротивлений характерны следующие свойства:

Напряжения на зажимах всех сопротивлений одинаковы:

U 1 = U 2 =U 3 =U ;

Проводимость всех параллельно соединённых сопротивлений равна сумме проводимостей отдельных сопротивлений:

1/R = 1/R 1 + 1/R 2 + 1/R 3 = R 1 R 2 + R 1 R 3 + R 2 R 3 /R 1 R 2 R 3 ,

где R — эквивалентное (равнодействующее) сопротивление трёх сопротивлений (в данном случае R 1 , R 2 и R 3 ) .

Чтобы получить сопротивление такой цепи, надо перевернуть дробь, определяющую величину её проводимости. Следовательно, сопротивление параллельного разветвления из трёх резисторов:

R = R 1 R 2 R 3 /R 1 R 2 + R 2 R 3 + R 1 R 3 .

Эквивалентным сопротивлением называется такое сопротивление, которым можно заменить несколько сопротивлений (включенных параллельно или последовательно), не изменяя величины тока в цепи.

Чтобы найти эквивалентное сопротивление при параллельном соединении, необходимо сложить проводимости всех отдельных участков, т.е. найти общую проводимость. Величина, обратная общей проводимости, и является общим сопротивлением.

При параллельном соединении эквивалентная проводимость равна сумме проводимостей отдельных ветвей, следовательно, эквивалентное сопротивление в этом случае всегда меньше наименьшего из параллельно включенных сопротивлений.

На практике могут быть случаи, когда цепь состоит из более, чем трёх параллельных ветвей. Все полученные соотношения остаются справедливыми и для цепей, состоящих из любого числа параллельно соединённых резисторов.

Найдём эквивалентное сопротивление двух параллельно включенных сопротивлений R 1 и R 2 (см. рис.). Проводимость первой ветви равна 1/R 1 , проводимость второй ветви — 1/R 2 . Общая проводимость:

1/R = 1/R 1 + 1/R 2 .

Приведём к общему знаменателю:

1/R = R 2 + R 1 /R 1 R 2 ,

отсюда эквивалентное сопротивление

R = R 1 R 2 /R 1 + R 2 .

Эта формула и служит для расчётов общего сопротивления цепи, состоящей из двух параллельно включенных сопротивлений.

Таким образом, эквивалентное сопротивление двух параллельно включенных сопротивлений равно произведению этих сопротивлений, делённому на их сумму.

При параллельном соединении n равных сопротивлений R 1 эквивалентное сопротивление их будет в n раз меньше, т.е.

R = R 1 /n .

На схеме, изображённой на последнем рисунке, включено пять сопротивлений R 1 по 30 Ом каждое. Следовательно, общее сопротивление R будет

R = R 1 /5 = 30/5 = 6 Ом.

Можно сказать, что сумма токов, подходящих к узловой точке А (на первом рисунке), равна сумме токов, от неё отходящих:

I = I 1 + I 2 + I 3 .

Рассмотрим, как происходит разветвление тока в цепях с сопротивлениями R 1 и R 2 (второй рисунок). Так как напряжение на зажимах этих сопротивлений одинаково, то

U = I 1 R 1 и U = I 2 R 2 .

Левые части этих равенств одинаковы, следовательно, равны и правые части:

I 1 R 1 = I 2 R 2 ,

или

I 1 /I 2 = R 2 /R 1 ,

Т.е. ток при параллельном соединении сопротивлений разветвляется обратно пропорционально сопротивлениям ветвей (или прямо пропорционально их проводимостям). Чем больше сопротивление ветви, тем меньше ток в ней, и наоборот.

Таким образом, из нескольких одинаковых резисторов можно получить общий резистор с бОльшей мощностью рассеивания.

При параллельном соединении неодинаковых резисторов в наиболее высокоомном резисторе выделяется наибольшая мощность.

Пример 1. Имеются два сопротивления, включенных параллельно. Сопротивление R 1 = 25 Ом, а R 2 = 50 Ом. Определить общее сопротивление цепи R общ .

Решение. R общ = R 1 R 2 /R 1 + R 2 = 25 . 50 / 25 + 50 ≈ 16, 6 Ом.

Пример 2. В ламповом усилителе имеются три лампы, нити накала которых включены параллельно. Ток накала первой лампы I 1 = 1 ампер, второй I 2 = 1, 5 ампера и третьей I 3 = 2, 5 ампера. Определить общий ток цепи накала ламп усилителя I общ .

Решение. I общ = I 1 + I 2 + I 3 = 1 + 1, 5 + 2, 5 = 5 ампер.

Параллельное соединение резисторов часто встречается в радиотехнической аппаратуре. Два или более резисторов включается параллельно в тех случаях, когда ток в цепи слишком большой и может вызвать чрезмерный нагрев резистора.

Примером параллельного соединения потребителей электрической энергии может служить включение электрических ламп обычной осветительной сети, которые соединяются параллельно. Достоинство параллельного соединения потребителей заключается в том, что выключение одного из них не влияет на работу других.

Какие есть способы соединения приемников электрической энергии

Какие есть способы соединения приемников электрической энергии

Приемник электрической энергии — устройство, в котором происходит преобразование электрической энергии в другой вид энергии для ее использования.

При одновременном включении нескольких приемников электроэнергии в одну и ту же сеть, эти приемники можно легко рассматривать просто как элементы единой цепи, каждый из которых обладает собственным сопротивлением.

В ряде случаев такой подход оказывается вполне приемлемым: лампы накаливания, электрические обогреватели и т. п. — можно воспринимать как резисторы. То есть приборы можно заменить на их сопротивления, и легко произвести расчет параметров цепи.

Способ соединения приемников электроэнергии может быть одним из следующих: последовательный, параллельный или смешанный тип соединения.

Последовательное соединение

Когда несколько приемников (резисторов) соединяются в последовательную цепь, то есть второй вывод первого присоединяется к первому выводу второго, второй вывод второго соединяется с первым выводом третьего, второй вывод третьего с первым выводом четвертого и т. д., то при подключении такой цепи к источнику питания, через все элементы цепи потечет ток I одной и той же величины. Данную мысль поясняет приведенный рисунок.

Заменив приборы на их сопротивления, рисунок преобразуем в схему, тогда сопротивления с R1 по R4, соединенные последовательно, примут каждый на себя определенные напряжения, которые в сумме дадут значение ЭДС на зажимах источника питания. Для простоты здесь и далее изобразим источник в виде гальванического элемента.

Выразив падения напряжений через ток и через сопротивления, получим выражение для эквивалентного сопротивления последовательной цепи приемников: общее сопротивление последовательного соединения резисторов всегда равно алгебраической сумме всех сопротивлений, составляющих эту цепь. А поскольку напряжения на каждом из участков цепи можно найти из закона Ома (U = I*R, U1 = I*R1, U2 = I*R2 и т. д.) и E = U, то для нашей схемы получаем:

Напряжение на клеммах источника питания равно сумме падений напряжений на каждом из соединенных последовательно приемников, составляющих цепь.

Так как ток через всю цепь течет одного и того же значения, то справедливым будет утверждение, что напряжения на последовательно соединенных приемниках (резисторах) соотносятся между собой пропорционально сопротивлениям. И чем выше будет сопротивление, тем выше окажется и напряжение, приложенное к приемнику.

Для последовательного соединения резисторов в количестве n штук, обладающих одинаковыми сопротивлениями Rk, эквивалентное общее сопротивление цепи целиком будет в n раз больше каждого из этих сопротивлений: R = n*Rk. Соответственно и напряжения, приложенные к каждому из резисторов цепи будут между собой равны, и окажутся в n раз меньше напряжения, приложенного ко всей цепи: Uk = U/n.

Для последовательного соединения приемников электроэнергии характерны следующие свойства: если изменить сопротивление одного из приемников цепи, то напряжения на остальных приемниках цепи при этом изменятся; при обрыве одного из приемников ток прекратится во всей цепи, во всех остальных приемниках.

В силу этих особенностей последовательное соединение встречается редко, и используют его лишь там, где напряжение сети выше номинального напряжения приемников, в отсутствие альтернатив.

К примеру напряжением 220 вольт можно запитать две последовательно соединенные лампы равной мощности, каждая из которых рассчитана на напряжение 110 вольт. Ежели данные лампы при одинаковом номинальном напряжении питания будут обладать различной номинальной мощностью, то одна из них будет перегружена и скорее всего мгновенно перегорит.

Параллельное соединение

Параллельное соединение приемников предполагает включение каждого из них между парой точек электрической цепи с тем, чтобы они образовывали параллельные ветви, каждая из которых питается напряжением источника. Для наглядности опять заменим приемники их электрическими сопротивлениями, чтобы получить схему, по которой удобно вести расчет параметров.

Как уже было сказано, в случае параллельного соединения каждый из резисторов испытывает действие одного и того же напряжения. И в соответствии с законом Ома имеем: I1=U/R1, I2=U/R2, I3=U/R3.

Здесь I – ток источника. Первый закон Кирхгофа для данной цепи позволяет записать выражение для тока в неразветвленной ее части: I = I1+I2+I3.

Отсюда общее сопротивление для параллельного соединения между собой элементов цепи можно найти из формулы:

Величина обратная сопротивлению называется проводимостью G, и формулу для проводимости цепи, состоящей из нескольких параллельно соединенных элементов, также можно записать: G = G1 + G2 + G3. Проводимость цепи в случае параллельного соединения образующих ее резисторов равна алгебраической сумме проводимостей этих резисторов. Следовательно, при добавлении в цепь параллельных приемников (резисторов) суммарное сопротивление цепи уменьшится, а суммарная проводимость соответственно возрастет.

Токи в цепи состоящей из параллельно соединенных приемников, распределяются между ними прямо пропорционально их проводимостям, то есть обратно пропорционально их сопротивлениям. Здесь можно привести аналогию из гидравлики, где поток воды распределяется по трубам в соответствии с их сечениями, тогда большее сечение аналогично меньшему сопротивлению, то есть большей проводимости.

Если цепь состоит из нескольких (n) одинаковых резисторов, соединенных параллельно, то общее сопротивление цепи будет ниже в n раз, чем сопротивление одного из резисторов, а ток через каждый из резисторов будет меньше в n раз, чем общий ток: R = R1/n; I1 = I/n.

Цепь, состоящая из параллельно соединенных приемников, подключенная к источнику питания, отличается тем, что каждый из приемников находится под напряжением источника питания.

Для идеального источника электроэнергии справедливо утверждение: при подключении или отключении параллельно источнику резисторов, токи в остальных подключенных резисторах не изменятся, то есть при выходе из строя одного или нескольких приемников параллельной цепи, остальные будут продолжать работать в прежнем режиме.

В силу данных особенностей параллельное соединение обладает значительным преимуществом перед последовательным, и по этой причине именно соединение параллельное наиболее распространено в электрических сетях. Например, все электроприборы в наших домах предназначены для параллельного подключения к бытовой сети, и если отключить один, то остальным это ничуть не навредит.

Сравнение последовательных и параллельных цепей

Смешанное соединение

Под смешанным соединением приемников понимают такое их соединение, когда часть или несколько из них соединены между собой последовательно, а другая часть или несколько — параллельно. При этом вся цепь может быть образована из разных соединений таких частей между собой. Для примера рассмотрим схему:

Три последовательно соединенных резистора подключены к источнику питания, параллельно одному из них подключены еще два, а третий — параллельно всей цепи. Для нахождения полного сопротивления цепи идут путем последовательных преобразований: сложную цепь последовательно приводят к простому виду, последовательно вычисляя сопротивление каждого звена, и так находят общее эквивалентное сопротивление.

Для нашего примера. Сначала находят общее сопротивление двух резисторов R4 и R5, соединенных последовательно, затем сопротивление параллельного соединения их с R2, потом прибавляют к полученному значению R1 и R3, и после — вычисляют значение сопротивления всей цепи, включая параллельную ветвь R6.

Различные способы соединения приемников электроэнергии применяют на практике для различных целей, чтобы решать конкретные поставленные задачи. Например, смешанное соединение можно встретить в схемах плавного заряда электролитических конденсаторов в мощных блоках питания, где нагрузка (конденсаторы после диодного моста) сначала получает питание последовательно через резистор, затем резистор шунтируется контактами реле, и нагрузка оказывается подключенной к диодному мосту параллельно.

Ранее ЭлектроВести писали, что львовянка Оксана Денис подключила свою квартиру к солнечным панелям и ветроэнергетическим установкам.

По материалам: electrik.info.

При параллельном соединении ток. Соединение резисторов параллельно и последовательно

Возьмем три постоянных сопротивления R1, R2 и R3 и включим их в цепь так, чтобы конец первого сопротивления R1 был соединен с началом второго сопротивления R 2, конец второго — с началом третьего R 3, а к началу первого сопротивления и к концу третьего подведем проводники от источника тока (рис. 1 ).

Такое соединение сопротивлений называется последовательным. Очевидно, что ток в такой цепи будет во всех ее точках один и тот же.

Рис 1

Как определить общее сопротивление цепи, если все включенные в нее последовательно сопротивления мы уже знаем? Используя положение, что напряжение U на зажимах источника тока равно сумме падений напряжений на участках цепи, мы можем написать:

U = U1 + U2 + U3

где

U1 = IR1 U2 = IR2 и U3 = IR3

или

IR = IR1 + IR2 + IR3

Вынеся в правой части равенства I за скобки, получим IR = I(R1 + R2 + R3) .

Поделив теперь обе части равенства на I , будем окончательно иметь R = R1 + R2 + R3

Таким образом, мы пришли к выводу, что при последовательном соединении сопротивлений общее сопротивление всей цепи равно сумме сопротивлений отдельных участков.

Проверим этот вывод на следующем примере. Возьмем три постоянных сопротивления, величины которых известны (например, R1 == 10 Ом, R 2 = 20 Ом и R 3 = 50 Ом). Соединим их последовательно (рис. 2 ) и подключим к источнику тока, ЭДС которого равна 60 В ( пренебрегаем).


Рис. 2. Пример последовательного соединения трех сопротивлений

Подсчитаем, какие показания должны дать приборы, включенные, как показано на схеме, если замкнуть цепь. Определим внешнее сопротивление цепи: R = 10 + 20 + 50 = 80 Ом.

Найдем ток в цепи : 60 / 80 = 0 ,75 А

Зная ток в цепи и сопротивления ее участков, определим падение напряжения на каждое участке цепи U 1 = 0,75х 10 = 7,5 В, U 2 = 0,75 х 20=15 В, U3 = 0,75 х 50 = 37,5 В.

Зная падение напряжений на участках, определим общее падение напряжения во внешней цепи, т. е. напряжение на зажимах источника тока U = 7,5+15 + 37,5 = 60 В.

Мы получили таким образом, что U = 60 В, т. е. несуществующее равенство ЭДС источника тока и его напряжения. Объясняется это тем, что мы пренебрегли внутренним сопротивлением источника тока.

Замкнув теперь ключ выключатель К, можно убедиться по приборам, что наши подсчеты примерно верны.

Возьмем два постоянных сопротивления R1 и R2 и соединим их так, чтобы начала этих сопротивлений были включены в одну общую точку а, а концы — в другую общую точку б. Соединив затем точки а и б с источником тока, получим замкнутую электрическую цепь. Такое соединение сопротивлений называется параллельным соединением.


Рис 3. Параллельное соединение сопротивлений

Проследим течение тока в этой цепи. От положительного полюса источника тока по соединительному проводнику ток дойдет до точки а. В точке а он разветвится, так как здесь сама цепь разветвляется на две отдельные ветви: первую ветвь с сопротивлением R1 и вторую — с сопротивлением R2. Обозначим токи в этих ветвях соответственно через I1 и I 2. Каждый из этих токов пойдет по своей ветви до точки б. В этой точке произойдет слияние токов в один общий ток, который и придет к отрицательному полюсу источника тока.

Таким образом, при параллельном соединении сопротивлений получается разветвленная цепь. Посмотрим, какое же будет соотношение между токами в составленной нами цепи.

Включим амперметр между положительным полюсом источника тока (+) и точкой а и заметим его показания. Включив затем амперметр (показанный «а рисунке пунктиром) в провод, соединяющий точку б с отрицательным полюсом источника тока (-), заметим, что прибор покажет ту же величину силы тока.

Значит, до ее разветвления (до точки а) равна силе тока после разветвления цепи (после точки б).

Будем теперь включать амперметр поочередно в каждую ветвь цепи, запоминая показания прибора. Пусть в первой ветви амперметр покажет силу тока I1 , а во второй — I 2. Сложив эти два показания амперметра, мы получим суммарный ток, по величине равный току I до разветвления (до точки а).

Следовательно, сила тока, протекающего до точки разветвления, равна сумме сил токов, утекающих от этой точки. I = I1 + I2 Выражая это формулой, получим

Это соотношение, имеющее большое практическое значение, носит название закона разветвленной цепи .

Рассмотрим теперь, каково будет соотношение между токами в ветвях.

Включим между точками а и б вольтметр и посмотрим, что он нам покажет. Во-первых, вольтметр покажет напряжение источника тока, так как он подключен, как это видно из рис. 3 , непосредственно к зажимам источника тока. Во-вторых, вольтметр покажет падения напряжений U1 и U2 на сопротивлениях R1 и R2, так как он соединен с началом и концом каждого сопротивления.

Следовательно, при параллельном соединении сопротивлений напряжение на зажимах источника тока равно падению напряжения на каждом сопротивлении.

Это дает нам право написать, что U = U1 = U2 ,

где U — напряжение на зажимах источника тока; U1 — падение напряжения на сопротивлении R1 , U2 — падение напряжения на сопротивлении R2. Вспомним, что падение напряжения на участке цепи численно равно произведению силы тока, протекающего через этот участок, на сопротивление участка U = IR .

Поэтому для каждой ветви можно написать: U1 = I1R1 и U2 = I2R2 , но так как U1 = U2, то и I1R1 = I2R2 .

Применяя к этому выражению правило пропорции, получим I1/ I2 = U2 / U1 т. е. ток в первой ветви будет во столько раз больше (или меньше) тока во второй ветви, во сколько раз сопротивление первой ветви меньше (или больше) сопротивления второй ветви.

Итак, мы пришли к важному выводу, заключающемуся в том, что при параллельном соединении сопротивлений общий ток цепи разветвляется на токи, обратно пропорциональные величинам сопротивлении параллельных ветвей. Иначе говоря, чем больше сопротивление ветви, тем меньший ток потечет через нее, и, наоборот, чем меньше сопротивление ветви, тем больший ток потечет через эту ветвь.

Убедимся в правильности этой зависимости на следующем примере. Соберем схему, состоящую из двух параллельно соединенных сопротивлений R1 и R 2, подключенных к источнику тока. Пусть R1 = 10 Ом, R2 = 20 Ом и U = 3 В.

Подсчитаем сначала, что покажет нам амперметр, включенный в каждую ветвь:

I1 = U / R1 = 3 / 10 = 0 ,3 А = 300 мА

I 2 = U / R 2 = 3 / 20 = 0,15 А = 150 мА

Общий ток в цепи I = I1 +I2 = 300 + 150 = 450 мА

Проделанный нами расчет подтверждает, что при параллельном соединении сопротивлений ток в цепи разветвляется обратно пропорционально сопротивлениям.

Действительно, R1 == 10 Ом вдвое меньше R 2 = 20 Ом, при этом I1 = 300 мА вдвое больше I2 = 150 мА. Общий ток в цепи I = 450 мА разветвился на две части так, что большая его часть (I1 = 300 мА) пошла через меньшее сопротивление (R1 = 10 Ом), а меньшая часть (R2 = 150 мА) -через большее сопротивление (R 2 = 20 Ом).

Такое разветвление тока в параллельных ветвях сходно с течением жидкости по трубам. Представьте себе трубу А, которая в каком-то месте разветвляется на две трубы Б и В различного диаметра (рис. 4). Так как диаметр трубы Б больше диаметра трубок В, то через трубу Б в одно и то же время пройдет больше воды, чем через трубу В, которая оказывает потоку воды большее сопротивление.

Рис. 4

Рассмотрим теперь, чему будет равно общее сопротивление внешней цепи, состоящей из двух параллельно соединенных сопротивлений.

Под этим общим сопротивлением внешней цепи надо понимать такое сопротивление, которым можно было бы заменить при данном напряжении цепи оба параллельно включенных сопротивления, не изменяя при этом тока до разветвления. Такое сопротивление называется эквивалентным сопротивлением.

Вернемся к цепи, показанной на рис. 3, и посмотрим, чему будет равно эквивалентное сопротивление двух параллельно соединенных сопротивлений. Применяя к этой цепи закон Ома, мы можем написать: I = U/R , где I — ток во внешней цепи (до точки разветвления), U — напряжение внешней цепи, R — сопротивление внешней цепи, т. е. эквивалентное сопротивление.

Точно так же для каждой ветви I1 = U1 / R1 , I2 = U2 / R2 , где I1 и I 2 — токи в ветвях; U1 и U2 — напряжение на ветвях; R1 и R2 — сопротивления ветвей.

По закону разветвленной цепи: I = I1 + I2

Подставляя значения токов, получим U / R = U1 / R1 + U2 / R2

Так как при параллельном соединении U = U1 = U2 , то можем написать U / R = U / R1 + U / R2

Вынеся U в правой части равенства за скобки, получим U / R = U (1 / R1 + 1 / R2 )

Разделив теперь обе части равенства на U , будем окончательно иметь 1 / R = 1 / R1 + 1 / R2

Помня, что проводимостью называется величина, обратная сопротивлению , мы можем сказать, что в полученной формуле 1 / R — проводимость внешней цепи; 1 / R1 проводимость первой ветви; 1 / R2- проводимость второй ветви.

На основании этой формулы делаем вывод: при параллельном соединении проводимость внешней цепи равна сумме проводимостей отдельных ветвей.

Следовательно, чтобы определить эквивалентное сопротивление включенных параллельно сопротивлений, надо определить проводимость цепи и взять величину, ей обратную.

Из формулы также следует, что проводимость цепи больше проводимости каждой ветви, а это значит, что эквивалентное сопротивление внешней цепи меньше наименьшего из включенных параллельно сопротивлений.

Рассматривая случай параллельного соединения сопротивлений, мы взяли наиболее простую цепь, состоящую из двух ветвей. Однако на практике могут встретиться случаи, когда цепь состоит из трех и более параллельных ветвей. Как же поступать в этих случаях?

Оказывается, все полученные нами соотношения остаются справедливыми и для цепи, состоящей из любого числа параллельно соединенных сопротивлений.

Чтобы убедиться в этом, рассмотрим следующий пример.

Возьмем три сопротивления R1 = 10 Ом, R2 = 20 Ом и R3 = 60 Ом и соединим их параллельно. Определим эквивалентное сопротивление цепи (рис. 5 ).


Рис. 5. Цепь с тремя параллельно соединенными сопротивлениями

Применяя для этой цепи формулу 1 / R = 1 / R1 + 1 / R2 , можем написать 1 / R = 1 / R1 + 1 / R2 + 1 / R3 и, подставляя известные величины, получим 1 / R = 1 / 10 + 1 / 20 + 1 / 60

Сложим эта дроби: 1/R = 10 / 60 = 1 / 6, т. е.. проводимость цепи 1 / R = 1 / 6 Следовательно, эквивалентное сопротивление R = 6 Ом.

Таким образом, эквивалентное сопротивление меньше наименьшего из включенных параллельно в цепь сопротивлений , т. е. меньше сопротивления R1.

Посмотрим теперь, действительно ли это сопротивление является эквивалентным, т. е. таким, которое могло бы заменить включенные параллельно сопротивления в 10, 20 и 60 Ом, не изменяя при этом силы тока до разветвления цепи.

Допустим, что напряжение внешней цепи, а следовательно, и напряжение на сопротивлениях R1, R2, R3 равно 12 В. Тогда сила токов в ветвях будет: I1 = U/R1 = 12 / 10 = 1 ,2 А I 2 = U/R 2 = 12 / 20 = 1 ,6 А I 3 = U/R1 = 12 / 60 = 0,2 А

Общий ток в цепи получим, пользуясь формулой I = I1 + I2 + I3 =1,2 + 0,6 + 0,2 = 2 А.

Проверим по формуле закона Ома, получится ли в цепи ток силой 2 А, если вместо трех параллельно включенных известных нам сопротивлений включено одно эквивалентное им сопротивление 6 Ом.

I = U / R = 12 / 6 = 2 А

Как видим, найденное нами сопротивление R = 6 Ом действительно является для данной цепи эквивалентным.

В этом можно убедиться и на измерительных приборах, если собрать схему с взятыми нами сопротивлениями, измерить ток во внешней цепи (до разветвления), затем заменить параллельно включенные сопротивления одним сопротивлением 6 Ом и снова измерить ток. Показания амперметра и в том и в другом случае будут примерно одинаковыми.

На практике могут встретиться также параллельные соединения, для которых рассчитать эквивалентное сопротивление можно проще, т. е. не определяя предварительно проводимостей, сразу найти сопротивление.

Например, если соединены параллельно два сопротивления R1 и R2 , то формулу 1 / R = 1 / R1 + 1 / R2 можно преобразовать так: 1/R = (R2 + R1) / R1 R2 и, решая равенство относительно R, получить R = R1 х R2 / (R1 + R2 ), т. е. при параллельном соединении двух сопротивлений эквивалентное сопротивление цепи равно произведению включенных параллельно сопротивлений, деленному на их сумму.

Знаете ли Вы, что такое мысленный эксперимент, gedanken experiment?
Это несуществующая практика, потусторонний опыт, воображение того, чего нет на самом деле. Мысленные эксперименты подобны снам наяву. Они рождают чудовищ. В отличие от физического эксперимента, который является опытной проверкой гипотез, «мысленный эксперимент» фокуснически подменяет экспериментальную проверку желаемыми, не проверенными на практике выводами, манипулируя логикообразными построениями, реально нарушающими саму логику путем использования недоказанных посылок в качестве доказанных, то есть путем подмены. Таким образом, основной задачей заявителей «мысленных экспериментов» является обман слушателя или читателя путем замены настоящего физического эксперимента его «куклой» — фиктивными рассуждениями под честное слово без самой физической проверки.
Заполнение физики воображаемыми, «мысленными экспериментами» привело к возникновению абсурдной сюрреалистической, спутанно-запутанной картины мира. Настоящий исследователь должен отличать такие «фантики» от настоящих ценностей.

Релятивисты и позитивисты утверждают, что «мысленный эксперимент» весьма полезный интрумент для проверки теорий (также возникающих в нашем уме) на непротиворечивость. В этом они обманывают людей, так как любая проверка может осуществляться только независимым от объекта проверки источником. Сам заявитель гипотезы не может быть проверкой своего же заявления, так как причина самого этого заявления есть отсутствие видимых для заявителя противоречий в заявлении.

Это мы видим на примере СТО и ОТО, превратившихся в своеобразный вид религии, управляющей наукой и общественным мнением. Никакое количество фактов, противоречащих им, не может преодолеть формулу Эйнштейна: «Если факт не соответствует теории — измените факт» (В другом варианте » — Факт не соответствует теории? — Тем хуже для факта»).

Максимально, на что может претендовать «мысленный эксперимент» — это только на внутреннюю непротиворечивость гипотезы в рамках собственной, часто отнюдь не истинной логики заявителя. Соответсвие практике это не проверяет. Настоящая проверка может состояться только в действительном физическом эксперименте.

Эксперимент на то и эксперимент, что он есть не изощрение мысли, а проверка мысли. Непротиворечивая внутри себя мысль не может сама себя проверить. Это доказано Куртом Гёделем.

При одновременном включении нескольких приемников электроэнергии в одну и ту же сеть, эти приемники можно легко рассматривать просто как элементы единой цепи, каждый из которых обладает собственным сопротивлением.

В ряде случаев такой подход оказывается вполне приемлемым: лампы накаливания, электрические обогреватели и т. п. — можно воспринимать как резисторы. То есть приборы можно заменить на их сопротивления, и легко произвести расчет параметров цепи.

Способ соединения приемников электроэнергии может быть одним из следующих: последовательный, параллельный или смешанный тип соединения.

Последовательное соединение

Когда несколько приемников (резисторов) соединяются в последовательную цепь, то есть второй вывод первого присоединяется к первому выводу второго, второй вывод второго соединяется с первым выводом третьего, второй вывод третьего с первым выводом четвертого и т. д., то при подключении такой цепи к источнику питания, через все элементы цепи потечет ток I одной и той же величины. Данную мысль поясняет приведенный рисунок.

Заменив приборы на их сопротивления, рисунок преобразуем в схему, тогда сопротивления с R1 по R4, соединенные последовательно, примут каждый на себя определенные напряжения, которые в сумме дадут значение ЭДС на зажимах источника питания. Для простоты здесь и далее изобразим источник в виде гальванического элемента.

Выразив падения напряжений через ток и через сопротивления, получим выражение для эквивалентного сопротивления последовательной цепи приемников: общее сопротивление последовательного соединения резисторов всегда равно алгебраической сумме всех сопротивлений, составляющих эту цепь. А поскольку напряжения на каждом из участков цепи можно найти из закона Ома (U = I*R, U1 = I*R1, U2 = I*R2 и т. д.) и E = U, то для нашей схемы получаем:

Напряжение на клеммах источника питания равно сумме падений напряжений на каждом из соединенных последовательно приемников, составляющих цепь.

Так как ток через всю цепь течет одного и того же значения, то справедливым будет утверждение, что напряжения на последовательно соединенных приемниках (резисторах) соотносятся между собой пропорционально сопротивлениям. И чем выше будет сопротивление, тем выше окажется и напряжение, приложенное к приемнику.

Для последовательного соединения резисторов в количестве n штук, обладающих одинаковыми сопротивлениями Rk, эквивалентное общее сопротивление цепи целиком будет в n раз больше каждого из этих сопротивлений: R = n*Rk. Соответственно и напряжения, приложенные к каждому из резисторов цепи будут между собой равны, и окажутся в n раз меньше напряжения, приложенного ко всей цепи: Uk = U/n.

Для последовательного соединения приемников электроэнергии характерны следующие свойства: если изменить сопротивление одного из приемников цепи, то напряжения на остальных приемниках цепи при этом изменятся; при обрыве одного из приемников ток прекратится во всей цепи, во всех остальных приемниках.

В силу этих особенностей последовательное соединение встречается редко, и используют его лишь там, где напряжение сети выше номинального напряжения приемников, в отсутствие альтернатив.

К примеру напряжением 220 вольт можно запитать две последовательно соединенные лампы равной мощности, каждая из которых рассчитана на напряжение 110 вольт. Ежели данные лампы при одинаковом номинальном напряжении питания будут обладать различной номинальной мощностью, то одна из них будет перегружена и скорее всего мгновенно перегорит.

Параллельное соединение

Параллельное соединение приемников предполагает включение каждого из них между парой точек электрической цепи с тем, чтобы они образовывали параллельные ветви, каждая из которых питается напряжением источника. Для наглядности опять заменим приемники их электрическими сопротивлениями, чтобы получить схему, по которой удобно вести расчет параметров.

Как уже было сказано, в случае параллельного соединения каждый из резисторов испытывает действие одного и того же напряжения. И в соответствии с законом Ома имеем: I1=U/R1, I2=U/R2, I3=U/R3.

Здесь I — ток источника. Первый закон Кирхгофа для данной цепи позволяет записать выражение для тока в неразветвленной ее части: I = I1+I2+I3.

Отсюда общее сопротивление для параллельного соединения между собой элементов цепи можно найти из формулы:

Величина обратная сопротивлению называется проводимостью G, и формулу для проводимости цепи, состоящей из нескольких параллельно соединенных элементов, также можно записать: G = G1 + G2 + G3. Проводимость цепи в случае параллельного соединения образующих ее резисторов равна алгебраической сумме проводимостей этих резисторов. Следовательно, при добавлении в цепь параллельных приемников (резисторов) суммарное сопротивление цепи уменьшится, а суммарная проводимость соответственно возрастет.

Токи в цепи состоящей из параллельно соединенных приемников, распределяются между ними прямо пропорционально их проводимостям, то есть обратно пропорционально их сопротивлениям. Здесь можно привести аналогию из гидравлики, где поток воды распределяется по трубам в соответствии с их сечениями, тогда большее сечение аналогично меньшему сопротивлению, то есть большей проводимости.

Если цепь состоит из нескольких (n) одинаковых резисторов, соединенных параллельно, то общее сопротивление цепи будет ниже в n раз, чем сопротивление одного из резисторов, а ток через каждый из резисторов будет меньше в n раз, чем общий ток: R = R1/n; I1 = I/n.

Цепь, состоящая из параллельно соединенных приемников, подключенная к источнику питания, отличается тем, что каждый из приемников находится под напряжением источника питания.

Для идеального источника электроэнергии справедливо утверждение: при подключении или отключении параллельно источнику резисторов, токи в остальных подключенных резисторах не изменятся, то есть при выходе из строя одного или нескольких приемников параллельной цепи, остальные будут продолжать работать в прежнем режиме.

В силу данных особенностей параллельное соединение обладает значительным преимуществом перед последовательным, и по этой причине именно соединение параллельное наиболее распространено в электрических сетях. Например, все электроприборы в наших домах предназначены для параллельного подключения к бытовой сети, и если отключить один, то остальным это ничуть не навредит.

Сравнение последовательных и параллельных цепей

Под смешанным соединением приемников понимают такое их соединение, когда часть или несколько из них соединены между собой последовательно, а другая часть или несколько — параллельно. При этом вся цепь может быть образована из разных соединений таких частей между собой. Для примера рассмотрим схему:

Три последовательно соединенных резистора подключены к источнику питания, параллельно одному из них подключены еще два, а третий — параллельно всей цепи. Для нахождения полного сопротивления цепи идут путем последовательных преобразований: сложную цепь последовательно приводят к простому виду, последовательно вычисляя сопротивление каждого звена, и так находят общее эквивалентное сопротивление.

Для нашего примера. Сначала находят общее сопротивление двух резисторов R4 и R5, соединенных последовательно, затем сопротивление параллельного соединения их с R2, потом прибавляют к полученному значению R1 и R3, и после — вычисляют значение сопротивления всей цепи, включая параллельную ветвь R6.

Различные способы соединения приемников электроэнергии применяют на практике для различных целей, чтобы решать конкретные поставленные задачи. Например, смешанное соединение можно встретить в схемах плавного заряда в мощных блоках питания, где нагрузка (конденсаторы после диодного моста) сначала получает питание последовательно через резистор, затем резистор шунтируется контактами реле, и нагрузка оказывается подключенной к диодному мосту параллельно.

Андрей Повный

Параллельное и последовательное соединение проводников – способы коммутации электрической цепи. Электрические схемы любой сложности можно представить посредством указанных абстракций.

Определения

Существует два способа соединения проводников, становится возможным упростить расчет цепи произвольной сложности:

  • Конец предыдущего проводника соединен непосредственно с началом следующего — подключение называют последовательным. Образуется цепочка. Чтобы включить очередное звено, нужно электрическую схему разорвать, вставив туда новый проводник.
  • Начала проводников соединены одной точкой, концы – другой, подключение называется параллельным. Связку принято называть разветвлением. Каждый отдельный проводник образует ветвь. Общие точки именуются узлами электрической сети.

На практике чаще встречается смешанное включение проводников, часть соединена последовательно, часть – параллельно. Нужно разбить цепь простыми сегментами, решать задачу для каждого отдельно. Сколь угодно сложную электрическую схему можно описать параллельным, последовательным соединением проводников. Так делается на практике.

Использование параллельного и последовательного соединения проводников

Термины, применяемые к электрическим цепям

Теория выступает базисом формирования прочных знаний, немногие знают, чем напряжение (разность потенциалов) отличается от падения напряжения. В терминах физики внутренней цепью называют источник тока, находящееся вне – именуется внешней. Разграничение помогает правильно описать распределение поля. Ток совершает работу. В простейшем случае генерация тепла согласно закону Джоуля-Ленца. Заряженные частицы, передвигаясь в сторону меньшего потенциала, сталкиваются с кристаллической решеткой, отдают энергию. Происходит нагрев сопротивлений.

Для обеспечения движения нужно на концах проводника поддерживать разность потенциалов. Это называется напряжением участка цепи. Если просто поместить проводник в поле вдоль силовых линий, ток потечет, будет очень кратковременным. Процесс завершится наступлением равновесия. Внешнее поле будет уравновешено собственным полем зарядов, противоположным направлением. Ток прекратится. Чтобы процесс стал непрерывным, нужна внешняя сила.

Таким приводом движения электрической цепи выступает источник тока. Чтобы поддерживать потенциал, внутри совершается работа. Химическая реакция, как в гальваническом элементе, механические силы – генератор ГЭС. Заряды внутри источника движутся в противоположную полю сторону. Над этим совершается работа сторонних сил. Можно перефразировать приведенные выше формулировки, сказать:

  • Внешняя часть цепи, где заряды движутся, увлекаемые полем.
  • Внутренняя часть цепи, где заряды движутся против напряженности.

Генератор (источник тока) снабжен двумя полюсами. Обладающий меньшим потенциалом называется отрицательным, другой – положительным. В случае переменного тока полюсы непрерывно меняются местами. Непостоянно направление движения зарядов. Ток течет от положительного полюса к отрицательному. Движение положительных зарядов идет в направлении убывания потенциала. Согласно этому факту вводится понятие падения потенциала:

Падением потенциала участка цепи называется убыль потенциала в пределах отрезка. Формально это напряжение. Для ветвей параллельной цепи одинаково.

Под падением напряжения понимается и нечто иное. Величина, характеризующая тепловые потери, численно равна произведению тока на активное сопротивление участка. Законы Ома, Кирхгофа, рассмотренные ниже, формулируются для этого случая. В электрических двигателях, трансформаторах разница потенциалов может значительно отличаться от падения напряжения. Последнее характеризует потери на активном сопротивлении, тогда как первое учитывает полную работу источника тока.

При решение физических задач для упрощения двигатель может включать в свой состав ЭДС, направление действия которой противоположно эффекту источника питания. Учитывается факт потери энергии через реактивную часть импеданса. Школьный и вузовский курс физики отличается оторванностью от реальности. Вот почему студенты, раскрыв рот, слушают о явлениях, имеющих место в электротехнике. В период, предшествующий эпохе промышленной революции, открывались главные законы, ученый должен объединять роль теоретика и талантливого экспериментатора. Об этом открыто говорят предисловия к трудам Кирхгофа (работы Георга Ома на русский язык не переведены). Преподаватели буквально завлекали люд дополнительными лекциями, сдобренными наглядными, удивительными экспериментами.

Законы Ома и Кирхгофа применительно к последовательному и параллельному соединению проводников

Для решения реальных задач используются законы Ома и Кирхгофа. Первый выводил равенство чисто эмпирическим путем – экспериментально – второй начал математическим анализом задачи, потом проверил догадки практикой. Приведем некоторые сведения, помогающие решению задачи:

Посчитать сопротивления элементов при последовательном и параллельном соединении

Алгоритм расчета реальных цепей прост. Приведем некоторые тезисы касательно рассматриваемой тематики:

  1. При последовательном включении суммируются сопротивления, при параллельном — проводимости:
    1. Для резисторов закон переписывается в неизменной форме. При параллельном соединении итоговое сопротивление равняется произведению исходных, деленному на общую сумму. При последовательном – номиналы суммируются.
    2. Индуктивность выступает реактивным сопротивлением (j*ω*L), ведет себя, как обычный резистор. В плане написания формулы ничем не отличается. Нюанс, для всякого чисто мнимого импеданса, что нужно умножить результат на оператор j, круговую частоту ω (2*Пи*f). При последовательном соединении катушек индуктивности номиналы суммируются, при параллельном – складываются обратные величины.
    3. Мнимое сопротивление емкости записывается в виде: -j/ω*С. Легко заметить: складывая величины последовательного соединения, получим формулу, в точности как для резисторов и индуктивностей было при параллельном. Для конденсаторов все наоборот. При параллельном включении номиналы складываются, при последовательном – суммируются обратные величины.

Тезисы легко распространяются на произвольные случаи. Падение напряжения на двух открытых кремниевых диодах равно сумме. На практике составляет 1 вольт, точное значение зависит от типа полупроводникового элемента, характеристик. Аналогичным образом рассматривают источники питания: при последовательном включении номиналы складываются. Параллельное часто встречается на подстанциях, где трансформаторы ставят рядком. Напряжение будет одно (контролируются аппаратурой), делятся между ветвями. Коэффициент трансформации строго равен, блокируя возникновение негативных эффектов.

У некоторых вызывает затруднение случай: две батарейки разного номинала включены параллельно. Случай описывается вторым законом Кирхгофа, никакой сложности представить физику не может. При неравенстве номиналов двух источников берется среднее арифметическое, если пренебречь внутренним сопротивлением обоих. В противном случае решаются уравнения Кирхгофа для всех контуров. Неизвестными будут токи (всего три), общее количество которых равно числу уравнений. Для полного понимания привели рисунок.

Пример решения уравнений Кирхгофа

Посмотрим изображение: по условию задачи, источник Е1 сильнее, нежели Е2. Направление токов в контуре берем из здравых соображений. Но если бы проставили неправильно, после решения задачи один получился бы с отрицательным знаком. Следовало тогда изменить направление. Очевидно, во внешней цепи ток течет, как показано на рисунке. Составляем уравнения Кирхгофа для трех контуров, вот что следует:

  1. Работа первого (сильного) источника тратится на создание тока во внешней цепи, преодоление слабости соседа (ток I2).
  2. Второй источник не совершает полезной работы в нагрузке, борется с первым. Иначе не скажешь.

Включение батареек разного номинала параллельно является безусловно вредным. Что наблюдается на подстанции при использовании трансформаторов с разным передаточным коэффициентом. Уравнительные токи не выполняют никакой полезной работы. Включенные параллельно разные батарейки начнут эффективно функционировать, когда сильная просядет до уровня слабой.

Если нам надо, чтобы электроприбор работал, мы должны подключить его к . При этом ток должен проходить через прибор и возвращаться вновь к источнику, то есть цепь должна быть замкнутой.

Но подключение каждого прибора к отдельному источнику осуществимо, в основном, в лабораторных условиях. В жизни же приходится иметь дело с ограниченным количеством источников и довольно большим количеством потребителей тока. Поэтому создают системы соединений, позволяющие нагрузить один источник большим количеством потребителей. Системы при этом могут быть сколь угодно сложными и разветвленными, но в их основе лежит всего два вида соединения: последовательное и параллельное соединение проводников. Каждый вид имеет свои особенности, плюсы и минусы. Рассмотрим их оба.

Последовательное соединение проводников

Последовательное соединение проводников – это включение в электрическую цепь нескольких приборов последовательно, друг за другом. Электроприборы в данном случае можно сравнить с людьми в хороводе, а их руки, держащие друг друга – это провода, соединяющие приборы. Источник тока в данном случае будет одним из участников хоровода.

Напряжение всей цепи при последовательном соединении будет равно сумме напряжений на каждом включенном в цепь элементе. Сила тока в цепи будет одинакова в любой точке. А сумма сопротивлений всех элементов составит общее сопротивление всей цепи. Поэтому последовательное сопротивление можно выразить на бумаге следующим образом:

I=I_1=I_2=⋯=I_n ; U=U_1+U_2+⋯+U_n ; R=R_1+R_2+⋯+R_n ,

Плюсом последовательного соединения является простота сборки, а минусом – то, что если один элемент выйдет из строя, то ток пропадет во всей цепи. В такой ситуации неработающий элемент будет подобен ключу в выключенном положении. Пример из жизни неудобства такого соединения наверняка припомнят все люди постарше, которые украшали елки гирляндами из лампочек.

Если в такой гирлянде выходила из строя хотя бы одна лампочка, приходилось перебирать их все, пока не найдешь ту самую, перегоревшую. В современных гирляндах эта проблема решена. В них используют специальные диодные лампочки, в которых при перегорании сплавляются вместе контакты, и ток продолжает беспрепятственно проходить дальше.

Параллельное соединение проводников

При параллельном соединении проводников все элементы цепи подключаются к одной и той же паре точек, можно назвать их А и В. К этой же паре точек подключают источник тока. То есть получается, что все элементы подключены к одинаковому напряжению между А и В. В то же время ток как бы разделяется на все нагрузки в зависимости от сопротивления каждой из них.

Параллельное соединение можно сравнить с течением реки, на пути которой возникла небольшая возвышенность. Вода в таком случае огибает возвышенность с двух сторон, а потом вновь сливается в один поток. Получается островок посреди реки. Так вот параллельное соединение – это два отдельных русла вокруг острова. А точки А и В – это места, где разъединяется и вновь соединяется общее русло реки.

Напряжение тока в каждой отдельной ветви будет равно общему напряжению в цепи. Общий ток цепи будет складываться из токов всех отдельных ветвей. А вот общее сопротивление цепи при параллельном соединении будет меньше сопротивления тока на каждой из ветвей. Это происходит потому, что общее сечение проводника между точками А и В как бы увеличивается за счет увеличения числа параллельно подключенных нагрузок. Поэтому общее сопротивление уменьшается. Параллельное соединение описывается следующими соотношениями:

U=U_1=U_2=⋯=U_n ; I=I_1+I_2+⋯+I_n ; 1/R=1/R_1 +1/R_2 +⋯+1/R_n ,

где I — сила тока, U- напряжение, R – сопротивление, 1,2,…,n – номера элементов, включенных в цепь.

Огромным плюсом параллельного соединения является то, что при выключении одного из элементов, цепь продолжает функционировать дальше. Все остальные элементы продолжают работать. Минусом является то, что все приборы должны быть рассчитаны на одно и то же напряжение. Именно параллельным образом устанавливают розетки сети 220 В в квартирах. Такое подключение позволяет включать различные приборы в сеть совершенно независимо друг от друга, и при выходе их строя одного из них, это не влияет на работу остальных.

Нужна помощь в учебе?

Предыдущая тема: Расчёт сопротивления проводников и реостаты: формулы
Следующая тема:&nbsp&nbsp&nbspРабота и мощность тока

Facebook

Twitter

Вконтакте

Одноклассники

Google+

21.1 Последовательные и параллельные резисторы — College Physics: OpenStax

На рисунке 3 показаны резисторы , подключенные параллельно , подключенные к источнику напряжения. Резисторы включены параллельно, когда каждый резистор подключен непосредственно к источнику напряжения с помощью соединительных проводов с незначительным сопротивлением. Таким образом, к каждому резистору приложено полное напряжение источника.

Каждый резистор потребляет такой же ток, как если бы он один был подключен к источнику напряжения (при условии, что источник напряжения не перегружен).Например, автомобильные фары, радио и т. Д. Подключены параллельно, так что они используют полное напряжение источника и могут работать полностью независимо. То же самое и в вашем доме, или в любом другом здании. (См. Рис. 3 (b).)

Чтобы найти выражение для эквивалентного параллельного сопротивления [латекс] \ boldsymbol {R _ {\ textbf {p}}} [/ latex], давайте рассмотрим протекающие токи и то, как они связаны с сопротивлением. Поскольку каждый резистор в цепи имеет полное напряжение, токи, протекающие через отдельные резисторы, составляют [латекс] \ boldsymbol {I_1 = \ frac {V} {R_1}} [/ latex], [латекс] \ boldsymbol {I_2 = \ frac {V} {R_2}} [/ latex] и [latex] \ boldsymbol {I_3 = \ frac {V} {R_3}} [/ latex].Сохранение заряда подразумевает, что полный ток [латекс] \ boldsymbol {I} [/ latex], производимый источником, является суммой этих токов:

[латекс] \ boldsymbol {I =} [/ латекс] [латекс] \ boldsymbol {\ frac {V} {R_1}} [/ latex] [латекс] \ boldsymbol {+} [/ latex] [латекс] \ boldsymbol {\ frac {V} {R_2}} [/ latex] [латекс] \ boldsymbol {+} [/ latex] [латекс] \ boldsymbol {\ frac {V} {R_3}} [/ latex] [латекс] \ boldsymbol {= V} [/ latex] [латекс] \ boldsymbol {(\ frac { 1} {R_1}} [/ latex] [латекс] \ boldsymbol {+} [/ latex] [латекс] \ boldsymbol {\ frac {1} {R_2}} [/ latex] [латекс] \ boldsymbol {+} [ / latex] [латекс] \ boldsymbol {\ frac {1} {R_3})}.[/ latex]

[латекс] \ boldsymbol {I =} [/ latex] [латекс] \ boldsymbol {\ frac {V} {R_p}} [/ latex] [латекс] \ boldsymbol {= V} [/ латекс ] [latex] \ boldsymbol {(\ frac {1} {R_p})}. [/ latex]

Члены в скобках в последних двух уравнениях должны быть равны. Обобщая для любого количества резисторов, общее сопротивление [латекс] \ boldsymbol {R_p} [/ latex] параллельного соединения связано с отдельными сопротивлениями соотношением

[латекс] \ boldsymbol {\ frac {1} {R_p}} [/ latex] [латекс] \ boldsymbol {=} [/ latex] [латекс] \ boldsymbol {\ frac {1} {R_1}} [/ latex] [латекс] \ boldsymbol {+} [/ latex] [латекс] \ boldsymbol {\ frac {1} {R_2}} [/ latex] [латекс] \ boldsymbol {+} [/ latex] [латекс] \ boldsymbol {\ frac {1} {R_3}} [/ latex] [латекс] \ boldsymbol {+ \ cdots} [/ latex]

Это соотношение приводит к общему сопротивлению [латекс] \ boldsymbol {R_p} [/ latex], которое меньше наименьшего из отдельных сопротивлений.(Это видно в следующем примере.) При параллельном подключении резисторов от источника течет больше тока, чем протекает по любому из них по отдельности, поэтому общее сопротивление ниже.

Пример 2: Расчет сопротивления, тока, рассеиваемой мощности и выходной мощности: анализ параллельной цепи

Пусть выходное напряжение батареи и сопротивления в параллельном соединении на рисунке 3 будут такими же, как и в ранее рассмотренном последовательном соединении: [латекс] \ boldsymbol {V = 12.0 \; \ textbf {V}} [/ latex], [latex] \ boldsymbol {R_1 = 1.00 \; \ Omega} [/ latex], [latex] \ boldsymbol {R_2 = 6.00 \; \ Omega} [/ латекс ] и [латекс] \ boldsymbol {R_3 = 13.0 \; \ Omega} [/ latex]. а) Каково полное сопротивление? (б) Найдите полный ток. (c) Рассчитайте токи в каждом резисторе и покажите, как они складываются, чтобы равняться общему выходному току источника. (d) Рассчитайте мощность, рассеиваемую каждым резистором. (e) Найдите выходную мощность источника и покажите, что она равна общей мощности, рассеиваемой резисторами.

Стратегия и решение для (а)

Общее сопротивление для параллельной комбинации резисторов находится с помощью следующего уравнения. Ввод известных значений дает

[латекс] \ boldsymbol {\ frac {1} {R_p}} [/ latex] [латекс] \ boldsymbol {=} [/ latex] [латекс] \ boldsymbol {\ frac {1} {R_1}} [/ латекс ] [латекс] \ boldsymbol {+} [/ латекс] [латекс] \ boldsymbol {\ frac {1} {R_2}} [/ latex] [латекс] \ boldsymbol {+} [/ латекс] [латекс] \ boldsymbol { \ frac {1} {R_3}} [/ latex] [latex] \ boldsymbol {=} [/ latex] [latex] \ boldsymbol {\ frac {1} {1.00 \; \ Omega}} [/ latex] [латекс] \ boldsymbol {+} [/ latex] [latex] \ boldsymbol {\ frac {1} {6.00 \; \ Omega}} [/ latex] [латекс] \ boldsymbol {+} [/ latex] [латекс] \ boldsymbol {\ frac {1} {13.0 \; \ Omega}}. [/ latex]

Таким образом,

[латекс] \ boldsymbol {\ frac {1} {R_p}} [/ latex] [латекс] \ boldsymbol {=} [/ latex] [латекс] \ boldsymbol {\ frac {1.00} {\ Omega}} [/ латекс] [латекс] \ boldsymbol {+} [/ латекс] [латекс] \ boldsymbol {\ frac {0.1667} {\ Omega}} [/ латекс] [латекс] \ boldsymbol {+} [/ латекс] [латекс] \ boldsymbol {\ frac {0.07692} {\ Omega}} [/ latex] [латекс] \ boldsymbol {=} [/ latex] [латекс] \ boldsymbol {\ frac {1.2436} {\ Omega}} [/ латекс]

(Обратите внимание, что в этих вычислениях каждый промежуточный ответ отображается с дополнительной цифрой.)

Мы должны перевернуть это, чтобы найти полное сопротивление [латекс] \ boldsymbol {R_p} [/ latex]. Это дает

[латекс] \ boldsymbol {R_p =} [/ latex] [латекс] \ boldsymbol {\ frac {1} {1.2436}} [/ latex] [латекс] \ boldsymbol {\ Omega = 0.8041 \; \ Omega}. [ / латекс]

Общее сопротивление с правильным количеством значащих цифр составляет [латекс] \ boldsymbol {R_p = 0.804 \; \ Omega} [/ latex]

Обсуждение для (а)

[латекс] \ boldsymbol {R_p} [/ latex], как и предполагалось, меньше минимального индивидуального сопротивления.

Стратегия и решение для (b)

Полный ток можно найти из закона Ома, заменив полное сопротивление [латекс] \ boldsymbol {R_p} [/ latex]. Это дает

[латекс] \ boldsymbol {I =} [/ latex] [латекс] \ boldsymbol {\ frac {V} {R_p}} [/ latex] [латекс] \ boldsymbol {=} [/ latex] [латекс] \ boldsymbol {\ frac {12.0 \; \ textbf {V}} {0.8041 \; \ Omega}} [/ latex] [latex] \ boldsymbol {= 14.92 \; \ textbf {A}} [/ latex]

Обсуждение для (б)

Ток [latex] \ boldsymbol {I} [/ latex] для каждого устройства намного больше, чем для тех же устройств, подключенных последовательно (см. Предыдущий пример).Схема с параллельным соединением имеет меньшее общее сопротивление, чем резисторы, включенные последовательно.

Стратегия и решение для (c)

Отдельные токи легко вычислить по закону Ома, поскольку каждый резистор получает полное напряжение. Таким образом,

[латекс] \ boldsymbol {I_1 =} [/ latex] [латекс] \ boldsymbol {\ frac {V} {R_1}} [/ latex] [латекс] \ boldsymbol {=} [/ latex] [латекс] \ boldsymbol {\ frac {12.0 \; \ textbf {V}} {1.00 \; \ Omega}} [/ latex] [latex] \ boldsymbol {= 12.0 \; \ textbf {A}}. [/ Latex]

Аналогично

[латекс] \ boldsymbol {I_2 =} [/ latex] [латекс] \ boldsymbol {\ frac {V} {R_2}} [/ latex] [латекс] \ boldsymbol {=} [/ latex] [латекс] \ boldsymbol {\ frac {12.0 \; \ textbf {V}} {6.00 \; \ Omega}} [/ latex] [latex] \ boldsymbol {= 2.00 \; \ textbf {A}} [/ latex]

и

[латекс] \ boldsymbol {I_3 =} [/ latex] [латекс] \ boldsymbol {\ frac {V} {R_3}} [/ latex] [латекс] \ boldsymbol {=} [/ latex] [латекс] \ boldsymbol {\ frac {12.0 \; \ textbf {V}} {13.0 \; \ Omega}} [/ latex] [latex] \ boldsymbol {= 0.92 \; \ textbf {A}}. [/ Latex]

Обсуждение для (c)

Общий ток складывается из отдельных токов:

[латекс] \ boldsymbol {I_1 + I_2 + I_3 = 14.92 \; \ textbf {A}}. [/ Latex]

Это соответствует сохранению заряда.

Стратегия и решение для (d)

Мощность, рассеиваемую каждым резистором, можно найти с помощью любого из уравнений, связывающих мощность с током, напряжением и сопротивлением, поскольку все три известны.2} {13.0 \; \ Omega}} [/ latex] [latex] \ boldsymbol {= 11.1 \; \ textbf {W}}. [/ Latex]

Обсуждение для (д)

Мощность, рассеиваемая каждым резистором параллельно, значительно выше, чем при последовательном подключении к тому же источнику напряжения.

Стратегия и решение для (e)

Общую мощность также можно рассчитать несколькими способами. Выбирая [латекс] \ boldsymbol {P = IV} [/ latex] и вводя общий ток, получаем

[латекс] \ boldsymbol {P = IV = (14.92 \; \ textbf {A}) (12.0 \; \ textbf {V}) = 179 \; \ textbf {W}}. [/ Latex]

Обсуждение для (e)

Суммарная мощность, рассеиваемая резисторами, также 179 Вт:

[латекс] \ boldsymbol {P_1 + P_2 + P_3 = 144 \; \ textbf {W} + 24.0 \; \ textbf {W} + 11.1 \; \ textbf {W} = 179 \; \ textbf {W}}. [/ латекс]

Это соответствует закону сохранения энергии.

Общее обсуждение

Обратите внимание, что как токи, так и мощность при параллельном подключении больше, чем для тех же устройств, подключенных последовательно.

Как рассчитать полное сопротивление параллельной цепи?

Как рассчитать полное сопротивление параллельной цепи?

Эффективное сопротивление резисторов, подключенных параллельно

  1. Есть три важных характеристики в параллельной цепи :
    (a) Разность потенциалов одинакова на каждом резисторе.
    (b) Ток, который проходит через каждый резистор, обратно пропорционален сопротивлению резистора.
    (c) Полный ток в цепи равен сумме токов, проходящих через резисторы в ее параллельных ветвях.
  2. Когда два или более сопротивления подключены между двумя общими точками так, что к каждой из них приложена одинаковая разность потенциалов, они считаются подключенными параллельно.
    Когда такая комбинация сопротивлений подключена к батарее, все сопротивления имеют одинаковую разность потенциалов на концах.
  3. Вывод математического выражения параллельной комбинации:
    Пусть, V будет разностью потенциалов между двумя общими точками A и B.Тогда по закону Ома
    Ток, проходящий через R 1 , I 1 = V / R 1 … (i)
    Ток, проходящий через R 2 , I 2 = V / R 2 … (ii)
    Ток, проходящий через R 3 , I 3 = V / R 3 … (iii)
  4. Если R — эквивалентное сопротивление, то по закону Ома полный ток, протекающий по цепи, определяется выражением
    I = V / R… (iv)
    и I = I 1 + I 2 + I 3 … (v)
  5. Подставляя значения I, I 1 , I 2 и I 3 в уравнение.(v),
    \ (\ frac {\ text {V}} {\ text {R}} = \ frac {\ text {V}} {{{\ text {R}} _ {\ text {1}} }} + \ frac {\ text {V}} {{{\ text {R}} _ {\ text {2}}}} + \ frac {\ text {V}} {{{\ text {R}} _ {\ text {3}}}} \ text {} …… .. \ text {(vi)} \)
  6. Отменяя общий термин V, получаем
    \ (\ frac {\ text {1}} {\ text {R}} = \ frac {\ text {1}} {{{\ text {R}} _ {\ текст {1}}}} + \ frac {\ text {1}} {{{\ text {R}} _ {2}}} + \ frac {\ text {1}} {{{\ text {R} } _ {3}}} \)
    Эквивалентное сопротивление параллельной комбинации сопротивлений меньше, чем каждое из всех отдельных сопротивлений.
  7. Эквивалентная схема показана на рисунке.

Важные результаты о параллельной комбинации:

  1. Полный ток в цепи равен сумме протекающих по ней токов.
  2. В параллельной комбинации резисторов напряжение (или разность потенциалов) на каждом резисторе одинакова и равна приложенному напряжению, то есть V 1 = V 2 = V 3 = V.
  3. Ток, протекающий через каждый резистор, обратно пропорционален его сопротивлению, поэтому чем выше сопротивление резистора, тем меньше ток, протекающий через него.

Люди также спрашивают

Проблемы параллельной цепи с решениями
  1. Три резистора, R 1 , R 2 и R 3 , подключены параллельно к батарее, как показано на рисунке.

    Рассчитайте
    (a) разность потенциалов на каждом резисторе,
    (b) эффективное сопротивление, R цепи,
    (c) ток, I, в цепи,
    (d) токи, I 1 , I 2 и I 3 , проходящие через каждый резистор.
    Решение:
    (a) Поскольку это параллельная схема, разность потенциалов на каждом резисторе составляет 6 В, то же самое, что и разность потенциалов на батарее, которая составляет 6 В.

Рассчитайте сопротивление цепи резистора онлайн. Последовательное соединение проводников. Параллельное соединение разных сопротивлений

Определение параллельного соединения

Параллельное соединение Электрические элементы (проводники, сопротивления, конденсаторы, индуктивности) — это такое соединение, при котором соединенные элементы цепи имеют две общие точки соединения.

Другое определение: сопротивления подключаются параллельно, если они подключены к одной и той же паре узлов.

Графическое обозначение схемы параллельного подключения

На рисунке ниже представлена ​​схема параллельного включения сопротивлений R1, R2, R3, R4. Из схемы видно, что все эти четыре сопротивления имеют две общие точки (точки подключения).

В электротехнике принято, но не обязательно, протягивать провода горизонтально и вертикально.Поэтому можно изобразить ту же схему, что и на рисунке ниже. Это тоже параллельное соединение одинаковых сопротивлений.

Формула для расчета параллельного соединения сопротивлений

При параллельном подключении величина, обратная эквивалентному сопротивлению, равна сумме обратных величин всех параллельно соединенных сопротивлений. Эквивалентная проводимость равна сумме всех одновременно подключенных проводимостей. электрическая цепь.


Для приведенной выше схемы эквивалентное сопротивление можно рассчитать по формуле:


В частном случае, когда два резистора соединены параллельно:

Сопротивление эквивалентной цепи определяется по формуле:

В случае подключения «n» того же сопротивления, эквивалентное сопротивление можно рассчитать по специальной формуле:

Формулы для частного расчета выводятся из основной формулы.

Формула для расчета параллельного включения конденсаторов (конденсаторов)

При параллельном соединении конденсаторов (конденсаторов) эквивалентная емкость равна сумме параллельно соединенных конденсаторов:

Формула для расчета индуктивностей параллельного включения

При параллельном соединении индуктивностей эквивалентная индуктивность рассчитывается так же, как эквивалентное сопротивление при параллельном соединении:


Следует обратить внимание, что взаимные индуктивности в формуле не учитываются.

Пример коагуляции параллельного сопротивления

Для электрической схемы участка необходимо найти параллельное соединение сопротивлений, чтобы преобразовать их в единицу.

Из схемы видно, что параллельно подключены только R2 и R4. R3 не параллелен, потому что на одном конце он подключен к E1. R1 — один конец подключен к R5, а не к узлу. R5 — одним концом подключен к R1, а не к узлу. Также можно сказать, что последовательное соединение сопротивлений R1 и R5 подключено параллельно с R2 и R4.

Параллельный ток

При параллельном соединении сопротивлений ток через каждое сопротивление обычно разный. Величина тока обратно пропорциональна величине сопротивления.

Напряжение при параллельном подключении

При параллельном включении разность потенциалов между узлами, соединяющими элементы схемы, одинакова для всех элементов.

Приложение для параллельного подключения

1. В промышленности изготавливаются сопротивления определенных количеств.Иногда необходимо получить значение сопротивления вне этой серии. Для этого можно параллельно подключить несколько сопротивлений. Эквивалентное сопротивление всегда будет меньше наибольшего значения сопротивления.

2. Делитель тока.

Параллельное соединение резисторов — один из двух типов электрических соединений, когда оба вывода одного резистора соединены с соответствующими выводами другого резистора или резисторов. Часто или параллельно с целью создания более сложных электронных схем.

Схема параллельного подключения показана на рисунке ниже. При параллельном соединении резисторов напряжение на всех резисторах будет одинаковым, а ток, протекающий через них, будет пропорционален их сопротивлению:

Формула для параллельного включения резисторов

Суммарное сопротивление нескольких параллельно включенных резисторов определяется по следующей формуле:

Ток, протекающий через одиночный резистор, согласно, можно найти по формуле:

Параллельное соединение резисторов — расчет

Пример №1

При разработке устройства возникла необходимость в установке резистора сопротивлением 8 Ом.Если просмотреть весь номинальный ряд стандартных номиналов резисторов, то мы увидим, что в нем нет резистора с сопротивлением 8 Ом.

Выход из этой ситуации — использование двух параллельно включенных резисторов. Эквивалентное значение сопротивления для двух параллельно включенных резисторов рассчитывается следующим образом:

Это уравнение показывает, что если R1 равен R2, то сопротивление R составляет половину сопротивления одного из двух резисторов. Следовательно, когда R = 8 Ом, R1 и R2 должны иметь значение 2 × 8 = 16 Ом.
Теперь проверим, рассчитав полное сопротивление двух резисторов:

Таким образом, мы получили необходимое сопротивление 8 Ом, подключив параллельно два резистора по 16 Ом.

Пример расчета № 2

Найдите полное сопротивление R трех параллельно соединенных резисторов:

Суммарное сопротивление R рассчитывается по формуле:

Этот метод расчета можно использовать для расчета любого количества отдельных сопротивлений, подключенных параллельно.

Один важный момент, который необходимо помнить при расчете параллельно соединенных резисторов, заключается в том, что полное сопротивление всегда будет меньше, чем значение наименьшего сопротивления в этой комбинации.

Как рассчитать схемы подключения комплексного резистора

Более сложные соединения резисторов можно рассчитать путем систематической группировки резисторов. На рисунке ниже нужно рассчитать полное сопротивление цепи, состоящей из трех резисторов:



Для простоты вычислений сначала сгруппируем резисторы по параллельному и последовательному включению.
Резисторы R2 и R3 включены последовательно (группа 2). Они, в свою очередь, подключены параллельно резистору R1 (группа 1).

Резисторы группы последовательного подключения 2 рассчитываются как сумма сопротивлений R2 и R3:

В результате упрощаем схему в виде двух параллельных резисторов. Теперь общее сопротивление всей цепи можно рассчитать следующим образом:

Расчет более сложных соединений резисторов можно выполнить по законам Кирхгофа.

Ток, протекающий в цепи параллельно включенных резисторов

Полный ток I, протекающий в цепи параллельных резисторов, равен сумме отдельных токов, протекающих во всех параллельных ветвях, и ток в одной ветви не обязательно должен быть равен току в соседних ветвях.

Несмотря на параллельное соединение, на каждый резистор подается одинаковое напряжение. А поскольку значение сопротивления в параллельной цепи может быть разным, то величина тока, протекающего через каждый резистор, также будет отличаться (по определению закона Ома).

Рассмотрим это на примере двух параллельно соединенных резисторов. Ток, протекающий через каждый из резисторов (I1 и I2), будет отличаться друг от друга, поскольку сопротивления резисторов R1 и R2 не равны.
Однако мы знаем, что ток, который входит в цепь в точке «A», должен выходить из цепи в точке «B».

Первое правило Кирхгофа гласит: «Полный ток, исходящий из цепи, равен току, входящему в цепь.«

Таким образом, протекающий полный ток в цепи можно определить как:

Затем, используя закон Ома, мы можем вычислить ток, протекающий через каждый резистор:

Ток протекающий через R1 = U ÷ R1 = 12 ÷ 22 кОм = 0,545 мА

Ток, протекающий в R 2 = U ÷ R2 = 12 ÷ 47 кОм = 0,255 мА

Таким образом, общий ток будет равен:

I = 0,545 мА + 0,255 мА = 0,8 мА

Это также можно проверить с помощью закона Ома:

I = U ÷ R = 12 В ÷ 15 кОм = 0.8 мА (то же)

, где 15 кОм — общее сопротивление двух параллельно соединенных резисторов (22 кОм и 47 кОм)

И в заключение следует отметить, что большинство современных резисторов маркированы цветными полосами и их назначение можно встретить.

Параллельное соединение резисторов — онлайн калькулятор

Для быстрого расчета общего сопротивления двух или более резисторов, соединенных параллельно, вы можете использовать следующий онлайн-калькулятор:

Подвести итог

Когда два или более резистора соединены так, что оба вывода одного резистора подключены к соответствующим выводам другого резистора или резисторов, они считаются подключенными параллельно друг другу.Напряжение на каждом резисторе внутри параллельной комбинации одинаковое, но токи, протекающие через них, могут отличаться друг от друга в зависимости от сопротивления каждого резистора.

Эквивалент или полное сопротивление параллельной комбинации всегда будет меньше минимального сопротивления резистора, включенного в параллельное соединение.

В каждой цепи есть резистор, имеющий сопротивление электрическому току. Резисторы бывают двух типов: фиксированные и переменные. При разработке любой электрической схемы и ремонте электронных изделий часто бывает необходимо использовать резистор, имеющий необходимый номинал.

Несмотря на то, что для резисторов существуют различные значения , может случиться так, что не удастся найти нужный или даже один элемент не сможет обеспечить требуемый показатель.

Решением этой проблемы может стать использование последовательного и параллельного подключения. Прочитав эту статью, вы узнаете об особенностях расчета и выбора различных значений сопротивления.

Часто при изготовлении любого устройства используют резисторы, которые подключаются по последовательной схеме.Эффект от использования такого варианта сборки сводится к увеличению общего сопротивления цепи. Для этого варианта соединения элементов создаваемое ими сопротивление рассчитывается как сумма номинальных значений. Если сборка деталей выполняется по параллельной схеме, здесь нужно рассчитать сопротивление по формулам ниже.

К схеме параллельного соединения прибегают в ситуации, когда стоит задача уменьшить общее сопротивление, и, кроме того, увеличить мощность для группы элементов, соединенных параллельно, которая должна быть больше, чем при их раздельном соединении.

Расчет сопротивления

В случае соединения частей между собой по параллельной схеме для расчета общего сопротивления будет использоваться следующая формула:

R (всего) = 1 / (1 / R1 + 1 / R2 + 1 / R3 + 1 / Rn).

  • R1- R3 и Rn — резисторы, включенные параллельно.

Причем, если схема создана на основе всего двух элементов, то для определения общего номинального сопротивления используйте следующую формулу:

R (всего) = R1 * R2 / R1 + R2.

  • R (total) — полное сопротивление;
  • R1 и R2 — резисторы, включенные параллельно.

Универсальная схема расчета

Применительно к радиотехнике следует обратить внимание на одно важное правило: если элементы соединены между собой по параллельной схеме имеют одинаковый показатель , то для расчета общего номинала необходимо общее значение разделить на количество подключаемых узлов:

  • R (total) — суммарное значение сопротивления;
  • R — номинал резистора, включенного параллельно;
  • n — количество подключенных узлов.

Особое внимание следует обратить на то, что конечный показатель сопротивления в случае использования схемы параллельного подключения обязательно будет меньше по сравнению с номиналом любого элемента, подключенного к цепи.

Пример расчета

Для большей наглядности мы можем рассмотреть следующий пример: предположим, что у нас есть три резистора, номиналы которых составляют соответственно 100, 150 и 30 Ом. Если мы используем первую формулу для определения общей стоимости, мы получим следующее:

R (всего) = 1 / (1/100 + 1/150 + 1/30) = 1 / (0.01 + 0,007 + 0,03) = 1 / 0,047 = 21,28 Ом.

Если произвести несложные вычисления, можно получить следующее: для схемы, состоящей из трех частей, где наименьшее значение сопротивления составляет 30 Ом, результирующее значение номинала будет равно 21,28 Ом. Этот показатель будет меньше минимального номинала в цепочке почти на 30%.

Важные нюансы

Обычно для резисторов параллельное соединение применяется, когда стоит задача создать сопротивление большей мощности.Для ее решения потребуются резисторы, которые должны иметь одинаковое сопротивление и мощность. С помощью этой опции вы можете определить общую мощность как : мощность одного элемента должна быть умножена на общее количество всех резисторов, которые составляют цепь, соединенную друг с другом в соответствии с параллельной цепью.

Допустим, если мы используем пять резисторов номиналом 100 Ом и мощностью каждого 1 Вт, которые соединены между собой по параллельной схеме, то общее сопротивление будет равно 20 Ом, а мощность будет 5 Вт.

Если взять те же резисторы, но подключить их по последовательной схеме, то конечная мощность будет 5 Вт, а общий номинал будет равен 500 Ом.

Заключение

Параллельная разводка резисторов очень востребована по той причине, что часто возникает задача создания такого номинала, чего невозможно добиться с помощью простого параллельного подключения. При этом процедура вычисления этого параметра довольно сложная , где необходимо учитывать разные параметры.

Здесь важную роль играет не только количество подключаемых элементов, но и рабочие параметры резисторов — в первую очередь сопротивление и мощность. Если один из подключаемых элементов будет иметь несоответствующий индикатор, то это не решит эффективно проблему создания необходимого номинала в цепи.

Последовательное соединение это соединение двух или более резисторов в виде цепи, в которой каждый отдельный резистор соединен с другим отдельным резистором только в одной точке.

Общее сопротивление R всего

При таком подключении одинаковый электрический ток проходит через все резисторы. Чем больше элементов в данном участке электрической цепи, тем «труднее» току течь через него. Следовательно, при последовательном соединении резисторов их общее сопротивление увеличивается, и оно равно сумме всех сопротивлений.

Напряжение при последовательном подключении

Напряжение при последовательном подключении распределяется на каждый резистор по закону Ома:

Т.е. чем больше сопротивление резистора, тем больше на нем падает напряжение.

Параллельное соединение это соединение, при котором резисторы соединены друг с другом обоими контактами. В результате к одной точке (электрическому узлу) можно подключить несколько резисторов.

Полное сопротивление R всего

При таком подключении через каждый резистор будет протекать отдельный ток. Сила этого тока будет обратно пропорциональна сопротивлению резистора. В результате общая проводимость такой части электрической цепи увеличивается, а общее сопротивление, в свою очередь, уменьшается.

Таким образом, при параллельном соединении резисторов с разным сопротивлением общее сопротивление всегда будет меньше, чем значение наименьшего отдельного резистора.

Формула общей проводимости резисторов при параллельном включении:

Формула эквивалентного общего сопротивления для параллельного включения резисторов:

Для двух одинаковых резисторов общее сопротивление будет составлять половину от одного отдельного резистора:

Соответственно, для n одинаковых резисторов полное сопротивление будет равно значению одного резистора, деленному на n.

Напряжение при параллельном подключении

Напряжение между точками A и B — это как общее напряжение для всего участка цепи, так и напряжение, приходящееся на каждый резистор отдельно. Следовательно, при параллельном подключении ко всем резисторам будет падать одинаковое напряжение.

Через каждый резистор протекает ток, мощность которого обратно пропорциональна сопротивлению резистора. Чтобы узнать, какой ток протекает через конкретный резистор, можно воспользоваться законом Ома:

Смешанным соединением называется участок цепи, в котором некоторые резисторы подключены друг к другу последовательно, а некоторые — параллельно.В свою очередь смешанное подключение бывает последовательного и параллельного типов.

Полное сопротивление R всего

  • Цепь разделена на участки с только параллельным или только последовательным подключением.
  • Рассчитайте общее сопротивление для каждой отдельной области.
  • Рассчитайте общее сопротивление для всей смешанной составной схемы.

Существует также более быстрый способ расчета общего сопротивления смешанного состава. Можно в соответствии со схемой сразу записать формулу так:

  • Если резисторы соединены последовательно, сложите.
  • Если резисторы подключены параллельно — используйте символ «||».
  • Заменить формулу параллельного соединения, где символ «||» стоит.

Так будет искать Схему 1:

В этой теме можно привести множество примеров из нашей повседневной жизни, касающихся параллельного соединения сопротивлений. Параллельное соединение одинаковых сопротивлений — наглядный пример подключения люстры с n-м числом ламп и с одинаковым сопротивлением для каждой лампы \ рис.1 \\.

Если предположить, что в люстре, состоящей из нескольких ламп \ с одинаковым сопротивлением \ одна лампа перегорела, а другую мощность заменили лампой другой мощности — в этом случае подключение люстры будет иметь вид параллельное соединение с разным сопротивлением.

Какие еще примеры можно привести из практики — с параллельным соединением сопротивлений? Допустим, вы подключили в своей квартире три бытовых электроприбора через удлинитель:

  • плита электрическая;
  • стиральная машина;
  • телевидение.

Природа этого соединения будет сопротивление параллельного соединения разное по величине. То есть для каждого прибора сопротивление имеет свое значение.

Расчет сопротивления при параллельном подключении

Как уже было сказано, при параллельном включении проводятся расчеты сопротивлений:

  • с таким же сопротивлением;
  • с разным сопротивлением

, а также выполнены расчеты сопротивлений при смешанном включении резисторов, при последовательном и параллельном включении \\ для одной цепи \\.Расчет сопротивления для смешанных подключений резисторов, больше подходит для различных блок-схем:

  • аудиоаппаратура;
  • видеоаппаратура.

К этой теме расчеты для смешанных соединений отношения не имеют.

Параллельное соединение одинаковых сопротивлений

Представьте себе параллельное соединение, например, трех равных по величине сопротивлений \\ рис.2 \\, где R1 = R2 = R3 = 36 Ом \ сопротивление лампы накаливания мощностью 95 Вт. Напряжение 220 В подключается к двум узловым точкам \ A, B \.Необходимо рассчитать суммарное сопротивление всех трех ламп.

Для расчета общего сопротивления \ R total \ нам нужно 36 Ом разделить на количество сопротивлений. Решение простое, Rtotal = 12 Ом. То есть формула расчета таких вычислений выглядит так:

R итого = R / n

Параллельное соединение разных сопротивлений

Допустим, мы берем выборочно три резистора, сопротивление:

  • R1 = 20 Ом;
  • R2 = 40 Ом;
  • R3 = 10 Ом.

Необходимо определить полное сопротивление резисторов при параллельном включении. Для этого расчета мы используем формулу:

1 / R всего = 1 / R1 + 1 / R2 + 1 / R3.

Подставьте значения в формулу:

1 / R всего = 1/20 + 1/40 + 1/10 = 7/40 = 0,18

получаем: Rtotal = 1 / 0,18 = 5,5 Ом.

Последовательные и параллельные резисторы — Электрические цепи — WJEC — GCSE Physics (Single Science) Revision — WJEC

Последовательные резисторы

При последовательном соединении резисторов ток через каждый резистор одинаков.Другими словами, ток одинаков во всех точках последовательной цепи.

При последовательном соединении резисторов общее напряжение (или разность потенциалов) на всех резисторах равно сумме напряжений на каждом резисторе.

Другими словами, напряжения в цепи складываются с напряжением источника питания.

Общее сопротивление ряда последовательно включенных резисторов равно сумме всех отдельных сопротивлений.

В этой схеме действует следующее.

I 1 = I 2 = I 3

V T = V 1 + V 2 + V 3

и, R T 162 = R 1 + R 2 + R 3

Последовательное добавление компонентов увеличивает общее сопротивление в цепи.

Параллельное соединение резисторов

При параллельном подключении резисторов ток питания равен сумме токов, протекающих через каждый резистор.Токи в ветвях параллельной цепи складываются с током питания.

Когда резисторы соединены параллельно, они имеют одинаковую разность потенциалов. Любые параллельные компоненты имеют одинаковую разность потенциалов.

Это уравнение используется для расчета общего сопротивления двух параллельно включенных резисторов.

\ [\ frac {1} {R} = \ frac {1} {R} _ {1} + \ frac {1} {R} _ {2} \]

Для расчета общего сопротивления трех резисторов подключенные параллельно, мы добавляем в уравнение третий резистор (и так далее).

\ [\ frac {1} {R} = \ frac {1} {R} _ {1} + \ frac {1} {R} _ {2} + \ frac {1} {R} _ {3 } \]

Параллельное добавление компонентов снижает общее сопротивление в цепи.

Вопрос

Рассчитайте сопротивление этой параллельной комбинации.

Показать ответ

\ [\ frac {1} {R} = \ frac {1} {R} _ {1} + \ frac {1} {R} _ {2} + \ frac {1} {R} _ {3} \]

\ [\ frac {1} {R} = \ frac {1} {3} + \ frac {1} {6} + \ frac {1} {9} \]

R = 1,64 Ом

Общее сопротивление меньше, чем у наименьшего резистора.

Электрическое сопротивление в последовательных и параллельных сетях

Последовательное соединение

Общее сопротивление для резисторов, подключенных последовательно, можно рассчитать как

R = R 1 + R 2 + …. + R n (1)

где

R = сопротивление (Ом, Ом)

Пример — Последовательные резисторы

Три резистора 33 Ом , 33 Ом и 3 47 Ом подключен последовательно.Общее сопротивление можно рассчитать как

R = ( 33 Ом) + ( 33 Ом) + ( 47 Ом)

= 113 Ом

Параллельное соединение

Общее сопротивление для резисторов, подключенных параллельно, можно рассчитать как

1 / R = 1 / R 1 + 1 / R 2 + …. + 1 / R n (2)

Эквивалентное сопротивление 2 резисторов, соединенных параллельно, можно выразить как

R = R 1 R 2 / (R 1 + R 2 ) (3)

Пример — Параллельно резисторы

Три резистора 33 Ом , 33 Ом и 47 Ом подключены параллельно.Общее сопротивление можно рассчитать как

1 / R = 1 / ( 33 Ом ) + 1 / ( 33 Ом ) + 1 / (47 Ом )

= 0,082 (1 / Ом)

R = 1 / (0,082 Ом)

= 12,2 Ом

Если напряжение батареи 12 В — ток в цепи можно рассчитать по закону Ома

I = U / R

= (12 В) / (12.2 Ом)

= 0,98 ампера

Можно рассчитать ток через каждый резистор

I 1 = U / R 1 = (12 В) / (33 Ом) = 0,36 ампера

I 2 = U / R 2 = (12 В) / (33 Ом) = 0,36 ампер

I 3 = U / R 3 = (12 В) / ( 47 Ом) = 0,26 ампера

Резисторы, подключенные параллельно — Калькулятор

Сложите сопротивления до пяти параллельно подключенных резисторов и (необязательно) напряжение цепи.

Общее сопротивление и ток — и отдельные токи во всех резисторах — будут рассчитаны:

R 1 (Ом)

R 2 (Ом)

R 3 ( Ом)

R 4 (Ом )

R 5 (Ом)

Напряжение (В)

I 1 (амперы) 2000 (амперы)

I 3 (амперы)

I 4 (амперы)

I 5 (амперы)

R I (амперы)

Резисторы и сопротивление — AP Physics 2

Пояснение:

Поскольку все резисторы подключены параллельно, все они будут испытывать одинаковое падение напряжения, равное напряжению батареи.Кроме того, поскольку каждый резистор имеет одинаковое сопротивление, ток, протекающий через четыре резистора, будет одинаковым.

Ток, протекающий через каждый резистор, можно показать по закону Ома:

Также, полный ток цепи можно найти, отметив:

Кроме того, общее сопротивление можно рассчитать, заметив, что резисторы, включенные параллельно, складываются обратно пропорционально.

Поскольку все резисторы одинаковые, можно отметить, что:

Подключаем к предыдущему уравнению, мы видим, что:

Таким образом, если четыре резистора соединены параллельно, общий ток в цепи равен.А поскольку общий ток равномерно распределяется между четырьмя резисторами, мы видим, что:

Теперь, если мы рассмотрим случай с пятью резисторами, соединенными параллельно, мы увидим, что общее сопротивление составляет:

Следовательно, общий ток в цепи при параллельном добавлении пятого резистора составляет:

И еще раз, поскольку каждый из пяти резисторов подключен параллельно, общий ток будет равномерно разделен между ними, что даст:

Таким образом, мы видим, что ток через каждый резистор одинаков как до, так и после добавления пятого резистора.

Подводя итог, можно сказать, что падение напряжения на каждом резисторе одинаково, поскольку они подключены параллельно. Кроме того, поскольку каждый резистор имеет одинаковое сопротивление, ток через каждый из них одинаков. Добавление пятого резистора параллельно уменьшило общее сопротивление цепи и увеличило общий ток, протекающий по цепи. Но поскольку этот новый, больший ток равномерно распределяется между всеми резисторами, включая новый, который был добавлен, ток через каждый из резисторов не изменяется.

параллельных резисторов

Обзор

Резисторы в цепи могут быть подключены последовательно или параллельно. Общее сопротивление комбинации резисторов зависит как от их индивидуальных значений, так и от способа их подключения.

Параллельные резисторы

Сопротивления параллельны, когда каждый резистор подключен непосредственно к источнику напряжения путем соединения проводов с незначительным сопротивлением. Таким образом, к каждому резистору приложено полное напряжение источника.

Параллельные резисторы

Параллельное соединение резисторов.

Каждый резистор потребляет такой же ток, как если бы он был единственным резистором, подключенным к источнику напряжения. Это верно для схем в доме или квартире. Каждая розетка, подключенная к устройству («резистор»), может работать независимо, и ток не должен проходить через каждое устройство последовательно.

Закон Ома и параллельные резисторы

Каждый резистор в цепи имеет полное напряжение.Согласно закону Ома токи, протекающие через отдельные резисторы, равны $ I_1 = \ frac {V} {R_1} $, $ I_2 = \ frac {V} {R_2} $ и $ I_3 = \ frac {V} {R_3 } $. Сохранение заряда подразумевает, что полный ток равен сумме этих токов:

Резисторы параллельные

Три резистора, подключенных параллельно к батарее, и эквивалентное одиночное или параллельное сопротивление.

$ I = I_1 + I_2 + I_3. $

Подстановка выражений для отдельных токов дает:

$ I = \ frac {V} {R_1} + \ frac {V} {R_2} + \ frac {V} {R_3} $

или

$ I = V (\ frac {1} {R_1} + \ frac {1} {R_2} + \ frac {1} {R_3}) $

Это означает, что полное сопротивление в параллельной цепи равно сумме инверсии каждого отдельного сопротивления.Следовательно, для каждой цепи с числом $ n $ или параллельно подключенными резисторами

$ R_ {n \; (parallel)} = \ frac {1} {R_1} + \ frac {1} {R_2} + \ frac {1} {R_3} … + \ frac {1} {R_n}. $

Это соотношение приводит к общему сопротивлению, которое меньше наименьшего из отдельных сопротивлений. Когда резисторы подключены параллельно, от источника течет больше тока, чем протекает для любого из них по отдельности, поэтому общее сопротивление ниже.

Каждый резистор, включенный параллельно, имеет такое же полное напряжение источника, как и источник, но делит общий ток между ними.Примером может служить соединение двух лампочек в параллельную цепь с аккумулятором на 1,5 В. В последовательной цепи две лампочки будут вдвое менее тусклыми при подключении к одному источнику батареи. Однако, если бы две лампочки были подключены параллельно, они были бы столь же яркими, как если бы они были подключены к батарее по отдельности. Поскольку к обеим лампочкам подается одинаковое полное напряжение, батарея также разряжается быстрее, поскольку она по существу обеспечивает полную энергию обеими лампочками.

Добавить комментарий

Ваш адрес email не будет опубликован. Обязательные поля помечены *